Scenario Three

  • November 2019
  • PDF

This document was uploaded by user and they confirmed that they have the permission to share it. If you are author or own the copyright of this book, please report to us by using this DMCA report form. Report DMCA


Overview

Download & View Scenario Three as PDF for free.

More details

  • Words: 24,253
  • Pages: 57
Scenario 3 Contents •

• • • • • • • • •

• • • • • • • •

  

 

 

       

Hypertension o Physiology – RAA system o Drugs Complications and risk factors for hypertension - hersh Anatomy and histology of pancreas - laura Control of glucose: insulin and glucagon Diabetes pathophysiology Risk factors of diabetes Genetics of diabetes Diabetes sx and signs Diabetes diagnosis and investigations Management of diabetes o Annual review o Drugs Microvascular complications of diabetes Macrovascular complications of diabetes Atherosclerosis Peripheral vascular disease and foot ulcers Diabetic emergencies Psychosocial aspects of diabetes Economics and diabetes Economics of chronic disease o Expert patient programmes Economics and epidemiology of obesity Metabolic Syndrome Malnutrition and nutrition 1. Water 2. Food 3. Fat Soluble Vitamins 4. Water Soluble Vitamins 5. Carbohydrates 6. Protein Synthesis 7. Fat lysis HONK and Ketoacidosis Nephrotoxic Drugs Glome Nephritis eGFR Ischaemic Foot Paul MDT diabetes Infections immune compromised Tests diabetes BMI Obesity Anatomy of Glom CKD

Blood Pressure Control of blood pressure The mean arterial pressure is determined by two factors: cardiac output and total peripheral resistance • Cardiac output depends on heart rate and stroke volume  Heart rate increases due to sympathetic nervous system and decreases by the action of parasympathetic nervous system  Stroke volume increases in response to sympathetic activity and it increases as venous return increases. Venous return is increased by sympathetically induced venous vasoconstriction, the skeletal muscle pump, the respiratory pump, cardiac suction and blood volume.  Blood volume depends on magnitude of passive bulk-flow fluid shifts between plasma and interstitial fluid across capillary walls and on the activity of the rennin-angiotensin-aldosterone system. • Total peripheral resistance depends on arteriolar radius and blood viscosity.  Blood viscosity depends mainly on number of red blood cells  Arteriolar radius is influenced by sympathetic activity that causes vasoconstriction and so increases total peripheral resistance and blood pressure. It is also influenced by local metabolic controls and hormones (vasopressin, angiotensin II) Barorecptor reflex is a very important short-term regulator of blood pressure that stimulates or inhibits sympathetic or parasympathetic activity accordingly. • Carotid sinus and aortic arch barorecptors are mechanoreceptors sensitive to changes in mean arterial and pulse pressure that trigger the baroreceptor reflex • Generated action potential are integrated in the cardiovascular control centre in medulla which alters accordingly the balance between sympathetic and parasympathetic activity to effector organs  Sympathetic stimulation increases heart rate and contractile strength of heart = cardiac output increases = blood pressure increases. It also causes arteriolar vasoconstriction and increases total peripheral resistance. It also causes venous vasoconstriction which increases venous return, stroke volume, cardiac output and blood pressure  Parasympathetic stimulation has the opposite effects. Rennin- angiotensin-aldosterone system is long-term regulator of blood pressure Rennin is an enzyme that is secreted by the juxtaglomerular apparatus in the following cases: 1. Low blood pressure leading to low renal perfusion pressure 2. A fall in Na concentration in distal tubule

3. Renal sympathetic nerve activity, β-adrenoreceptor agonist and PGI2 Angiotensin II and atrial natriuretic peptide inhibit rennin release. Rennin acts on angiotensinogen (made in liver) to produce angiotensin I which is converted to angiotensin II by angiotensin converting enzyme when it passes from lung. Angiotensin II has the following actions: 1. Vasoconstriction (direct and via increased noradrenaline release from sympathetic nerves) 2. Salt retention due to secretion of aldosterone and due to increased Na tubular reabsorption. 3. Vascular growth (hyperplasia and hypertrophy in heart and arteries) Hypertension • 95% of cases have essential hypertension with poorly understood mechanism • 5% of cases have secondary hypertension due to chronic renal parenchymal or vascular disease or a catecholamine or aldosterone-secreting adrenal tumour or due to drugs (NSAIDs, steroids, oral contraceptive pill). Pathophysiology • Increased peripheral vascular resistance due to widespread constriction of arterioles and small arteries. Cardiac output and viscosity of blood are normal. • In chronic hypertension structural changes take place that cause further hypertension or complications  High peripheral resistance = increased work of heart = left ventricular hypertrophy = hypertrophy outstrips coronary blood supply =fibrous tissue deposition= heart failure  Hypertension causes thickening of tunica media = arteries lose compliance = more pronounced pressure wave = mechanical stress and endothelial dysfunction = atherosclerosis  Changes in renal vasculature = reduced renal perfusion = reduced glomerular filtration rate = activation of rennin-angiotensin system = increase in blood pressure Risk factors • Genetic factors: polygenic definite influence of heredity and family history • Fetal factors: low birthweight possibly due to changes in blood vessel structure or hormonal changes • Environment: obesity, sleep disordered breathing, alcohol intake, sodium intake, stress • Diabetes is associated with hypertension. The described metabolic syndrome includes hyperinsulinemia, glucose intolerance, low HDL, hypertriglyceridaemia and central obesity, all of which are associated with insulin resistance. Investigations of hypertension • ECG (check for LV hypertrophy, ischemic heart disease) • Urinalysis for proteinuria if renal disease, vanillyl mandelic acid for malignant hypertension) • U&Es (high urea suggests renal impairment, low potassium without diuretic might suggest Conn’s (mineralocorticoid) or Cushing’s syndrome(corticosteroid) • Lipids (high cholesterol is cardiovascular risk factor) • 24-hour ambulatory blood pressure monitoring • Echocardiography (check for left ventricular hypertrophy) Management of hypertension • Smoking cessation, weight reduction, alcohol intake reduction, salt intake reduction, healthy diet (no saturated fats, more fruits and vegetables), regular physical exercise A= ace and alpha- B= bendroflumethiazide C= calcium channel- D= diuretic Young < 55 & non Black Older > 55 or Black Step 1 ACE inhibitor or B-blockers Calcium channel blockers or Diuretics Step 2 A (or B) + C or D C (or D) + A or B Step 3 A+C+D A+C+D Step 4 + alpha blocker or spironolactone + alpha blocker or spironolactone

• •

Statins to reduce cardiovascular risk in elderly with CHD, PVD, CVD or type II diabetes Low dose aspirin or clopidogrel if cardiovascular risk to reduce risk of strokes and MIs

Hypertension Causes of hypertension: BP >120/80mmHg. Essential HTN: No specific medical cause but associated with affecting 95% of hypertensives • Genetics (polygenetic) • Age • Lower birth weight • Obesity • Sleep apnoea • Alcohol • Sodium intake • Stress • Insulin resistance • High renin Secondary HTN: 5% of hypertensive’s where he cause is identified • Renal disease: PCK, chronic GN, diabetic • Endocrine: Conn’s syndrome, adrenal hyperplasia, pheochromocytoma, Cushing’s, acromegaly • Coarctation of aorta • Drugs: OCP, steroids, carbenoxolone, vasopressin, sudden withdrawal of antihypertensive

• • • • • •

PregnancyDamage caused by elevated BP Arteriolar thickening and more collagen in large arteries Decreased density of vessels Atheroma Left ventricular hypertrophy due to increased peripheral vascular resistance Secondary renal damage and activation of renin-angiotensin system which exacerbates hypertension

Investigations • CXR – cardiomegaly, pulmonary congestion • ECG/echocardiogram – coronary artery disease, LV hypertrophy • Urinalysis • Blood glucose and lipids (fasting) – DM and hypercholesterolemia • Serum U&Es, creatinine – if deranged indicates testing creatinine clearance, US, renal angiography. Hypokalemia points to an endocrine cause (hyperaldosteronism, glucocorticoid excess). Treatment see drugs table at end • Life-style: weight loss, low fat and low sodium diet, reduces alcohol, increase exercise, fruit and vegetable. • Control underlying diseases: blood sugar control, kidney diseases • Drug treatment: diuretics (reduce risk of stroke), beta-blockers (probably attenuate effects of sympathetic nervous and renin-angiotensin systems), ACE inhibitor or angiotensin II receptor antagonist (prevent vasoconstriction), Ca-channel blocker (cause arteriolar dilatation), alpha-blocker (vasodilation). NICE guidelines on when to refer • Immediately if: accelerated (malignant) hypertension (blood pressure more than 180/110 mmHg with papilloedema, retinal haemorrhage); suspected pheochromocytoma (labile or postural hypotension, headache, palpitations, pallor and diaphoresis). • Consider referral if: unusual signs and symptoms, some suggesting secondary cause; precise BP needed for management; symptoms of postural hypotension, or a fall in systolic blood pressure when standing of 20 mmHg or more. End organ damage Eyes



Hypertensive retinopathy. Changes: vasospastic reaction to an acute pressure rise, arteriosclerotic response to chronic elevation. Features include: • Generalised arteriolar narrowing (arteriosclerosis) through medial hyperplasia and fibrosis; broadening of arteriolar light reflex, venous nipping at AV crossing point. • Focal arteriolar narrowing: vasospastic effect during acute rise • Flame haemorrhages: in nerve fibre layer due to capillary damage. Also: dots and blots • Cotton wool spots: swollen axonal endings due to focal ischaemia • Exudates: intraretinal collections of lipids • Optic disc swelling: local ischaemia • Arteriolar macroaneurysms: HTN, atheroma • Microaneurysms Kidneys • HTN causes changes in intra-renal vasculature (nephrosclerosis) via: • Intimal thickening with reduplication of internal elastic lamina and hyalinisation of wall in small vessels and arterioles. • Concentric reduplication of internal elastic lamina and endothelial proliferation in large vessels • Reduction in size of kidneys in the long term • Increased proportion of sclerotic glomeruli • The result is microscopic haematuria, proteinuria, and progressive uraemia Heart • Systemic (L sided) hypertensive disease: left ventricular hypertrophy associated with HTN. Changes that are initially adaptive cause cardiac dilatation, CCF, sudden death. Heart adapts with LV hypertrophy. Long term this causes interstitial fibrosis, myocyte atrophyventricular dilatation. • Pulmonary hypertensive disease (cor pulmonale): right ventricular hypertrophy. Can be caused by sudden large PE or lung disease. There is chronic right ventricular pressure load. The RV dilatation my cause tricuspid regurgitation. Vascular • Accelerated atherosclerosis and narrowing of lumen • Arteries: muscular hypertrophy of media, reduplication of external lamina, intimal thickening • Arterioles: hyaline arteriosclerosis (protein deposits in wall) • Vessels in brain: microaneurysms (Charcot-Bouchard aneurysms) • Can be associated with fibrin deposits (necrotising arteriolitis) Management note B is now bendroflumethiazide not beta blocker (2008) Step 1 Step 2 Step 3 Step 4

Young < 55 & non Black ACE inhibitor A or B + C or D A+C+D + alpha blocker or spironolactone

Older > 55 or Black Calcium channel blockers or Diuretics C (or D) + A or B A+C+D + alpha blocker or spironolactone

Antihypertensives DRUG

ACE inhibitors

EXAMPLES Ramipril, Lisinopril

USES • • •

Hypertension Heart Failure Post MI

MECHANISM Block ATIATII by binding to the site on the enzyme that normally accommodates the terminal leucine of ATI. Inhibits vasoconstriction.

SIDE EFFECTS • • •

Beta-Blockers

Atenolol, Propanolol

• • • •

Hypertension Angina Arrhythmias Stable heart failure

block β-adrenergic receptors inhibiting the • effects of adrenaline and nonadrenaline. β1 (heart) blockage decreases HR and contractility, β2 (bronchial and vascular • • smooth muscle) blockage causes vasodilation.

Calcium Channel Blockers

Dihydropines – Amlodipine, Phenyalkylamin es Verapamil, Benzthiazepines Diltiazem

• • •

Hypertension Angina Supraventricul ar arrhythmia – (Phenylalkyla mines only)

Vasodilation block cellular entry of Ca+ by preventing opening of voltagegated L-type and T-type calcium channels

Thiazide Diuretics

Loop Diuretics

Potassiumsparing diuretics

Bendrofluazide, hydrochlorothiaz ide

Frusemide, Bumetanide

Spironolactone, Amiloride

• • •



• •



• • •

Hypertension Combined with loop for Heart Failure

Hypertension (but less effective than thiazides – used when renal impairment or resistant to multiple drug Tx) Heart Failure Secondary Hypertension Severe heart Failure

increase water excretion by decreasing reabsorption of Na+ and Cl- in the distal tubule by binding to the Cl- site of the electroneutral Na+/Cl- co-transport system and inhibiting its action causing a decrease in blood volume, venous return and CO Block Na+ resorption in ascending loop of Henle – diuretic effect.

• • • • • • • • • •

Amilioride directly blocks epithelial Na+ channel, spironolactone antagonises action of aldosterone in distal convoluted tubule – diuretic effect

• • •

Hypotension Dry cough (increased bradykinin) Renal failure in pts with bilateral renal stenosis Provocation of asthma, heart failure. Cold hands Bradycardia - fatigue Flushing, headache, P.oedema Phenyalkyla mines – can worsen heart failure Gynaecomas tia Impotence Hypokalaemi a Hyponatrae mia Hypotension Gout Type II DM Hypokalaemi a Hyponatrae mia Hypotension Gout

Hypokalaemi a Hyponatrae mia Abdominal discomfort

Angiotensin II receptor antagonists

Losarten, Valsarten

• •

Alphaadrenorecepto r antagonists

Doxazosin, Prazosin



Hypertension Alternative to ACE inhibitor in heart failure Hypertension (in addition to other hypertensives)

Vasodilation – by inhibition at the angiotensin II receptor

• •

Reduces peripheral resistance by inhibiting α1-adrenoreceptormediated vasoconstriction.

• •

Usually mild No cough like in ACE inhibitors Postural Hypotension Dizziness

Anatomy and Physiology of the Pancreas Anatomy of the pancreas • Consists of an uncinate process, a head slightly caudal, body and tail both slightly cranial. It is 12-15cm long and 2.5cm thick. • Location: o Lies in the transpyloric plane (L1) o It is retroperitoneal, crossed anteriorly by the mesentery of the transverse colon and lies posterior to the stomach separated by the lesser peritoneal sac o The head an the uncinate process lie anterior to the IVC within the loop of the duodenum on L1-L3 o The body passes obliquely to the left anterior to the aorta, left psoas, splenic artery and vein o The tail lies anterior to the left kidney and extends into the hilum of the spleen • It is connected to the duodenum by 2 ducts (sometimes they join to form a single duct): 1. The pancreatic duct (or duct of Wirsund) – joins with the common bile duct from the liver and gallbladder and enters the duodenum as a common duct called the hepatopancreatic ampulla (ampulla of Vater). The ampulla opens on an elevation of the duodenal mucosa known as major duodenal papilla which lies about 10cm inferior to the pyloric sphincter of the stomach. 2. The accessory duct (or duct of Santorini) – separate second branch of the pancreatic duct that empties into the duodenum usually in the major duodenal papilla with the pancreatic duct. However, rarely it empties via a separate opening about 2.5cm superior to the hepatopancreatic ampulla. • Blood supply and venous drainage: o Pancreatic head receives its supply from the superior and inferior pancreaticoduodenal arteries which come off the common hepatic artery and superior mesenteric artery respectively. o The splenic artery also supplies the pancreas and courses along its upper border. It provides a large branch called the arteria pancreatica magna and numerous smaller branches. o The venous drainage of the pancreas is via pancreatic and pancreaticoduodenal veins  splenic vein  superior mesenteric vein  hepatic portal vein  liver  hepatic veins  IVC • Innervation is via the pancreatic plexus, coeliac ganglia and vagus nerve. Histology • The pancreas is made up of clusters of glandular epithelial cells. • Endocrine pancreas: 1% of the cells are organised into clusters called pancreatic islets (islets of Langerhans) – these are the endocrine portion of the pancreas and secrete the hormones glucagon, insulin, somatostatin and pancreatic polypeptide. o Beta-cells (70%) – produce insulin o Alpha-cells (17%) – produce glucagon o Delta-cells (7%) – produce somatostatin o Pancreatic Polypeptide (PP) cells o Enterochromaffin cells – produce serotonin • Exocrine pancreas: 99% are arranged in clusters called acini – these constitute the exocrine portion of the organ. They secrete pancreatic juice. • Regulation of pancreatic secretions - under neural and hormonal control: 1. Parasympathetic impulses – transmitted along the vagus nerve (cranial nerve X) during cephalic and gastric phases of gastric digestion causing stimulation of pancreatic enzyme secretion. 2. Fatty acids and amino acids - entering the duodenum (acidic chyme) cause enteroendocrine cells to secrete:  Cholecystokinin (CCK) into the blood which stimulates pancreatic juice rich in digestive enzymes.  Secretin into the blood which stimulates pancreatic juice secretion rich in bicarbonate ions.

Exocrine function • Secretes pancreatic juice – clear colourless liquid consisting of water, some salts, sodium bicarbonate, and several enzymes. • Sodium bicarbonate gives the juice a slightly alkaline pH to buffer acidic chyme. This stops pepsin action and provides optimum pH for its own digestive enzymes. • The enzymes: Enzyme Inactive Form Enzyme Active Form Function Pancreatic amylase Digests carbohydrate Trypsinogen (activated by Trypsin Digests protein and enterokinase) activates inactive precursors Chymotrypsinogen Chymotrypsin (activated by trypsin) Procarboxypeptidase Carboxypeptidase (activated by trypsin) Proelastase (activated by Elastase trypsin) Pancreatic lipase Principle triglyceridedigesting enzyme Ribonuclease Nucleic-acid-digestin Deoxyribonuclease

enzymes

Endocrine function Insulin • Peptide hormone coded for on chromosome 11. It is made up of two polypeptide chains: A and B which are linked by disulphide bridges. • In beta-cells, DNA transcription produces pre-proinsulin, which is cleaved to proinsulin by the action of protease within the Golgi apparatus. Proinsulin is converted by convertase to insulin and a connecting peptide (c-peptide) which are released together in exocytosis. Secretion of insulin: • Secreted from β-cells into the portal vein in a biphasic pattern: first there is an acute, rapid phase followed by a less intense, more sustained second phase. • Stimulus for secretion:

o

Nutrients: glucose, amino acids

o Hormones: glucagon, gastrin, secretin, cholecystokinin, glucose-dependent insulinotropic peptide (GIP –





released by enteroendocrine cells of the small intestine in response to presence of glucose in the GIT), and indirectly ACTH and GH (both raise BGL) o Pancreatic innervation: sympathetic α receptors, parasympathetic (acetylcholine) Inhibitors of secretion: o Nutrients: fall in glucose o Hormones: somatostatin o Pancreatic innervation: sympathetic beta-receptors o Stress: exercise, hypoxia, hypothermia, surgery, severe burns Process of secretion of insulin: 1. Glucose enters the beta-cell via the GLUT-2 transporter protein (its rate of entry is proportional to its extracellular concentrations) 2. Glucose is phosphorylated to glucose-6-phosphate by the enzyme glucokinase. This enzyme is also the “sensor” of glucose in the beta-cell and stimulates insulin secretion. 3. ATP is released in the cascade (glycolysis) and ATP-sensitive K-channels in the beta-cell close (sulphonylureas binds to a receptor closely apposed to K-channels closing them). There is depolarisation of the cell and an influx of Ca2+ through ion channels. 4. Calcium sensitive proteins are activated and this triggers insulin translocation to the cell surface and exocytosis 5. Glucose is detected by the β-cell, it is taken up and undergoes phosphorylation and metabolism by glycolysis  release of ATP  closure of K-channels  triggers insulin granule

Insulin receptor binding: • Insulin binds to insulin receptors which are found on the surface of most body cells (the major insulin-sensitive tissues include: the liver, skeletal muscle and adipose tissue). • The insulin receptor is a glycoprotein (chromosome 19) which consists of 2 α subunits, binding sites for insulin and 2 β subunits which traverse the cell membrane. When insulin binds to the alpha subunits there is a conformational change in the beta subunits  activation of tyrosine kinase and initiation of a cascade response involving intracellular substrate proteins known as insulin responsive substrates (IRS). One consequence of this is migration of the GLUT-2 glucose transporter to the cell surface and increased transport of glucose into the cell. The insulin-receptor complex is then internalized by the cell, insulin is degraded, and the receptor is recycled to the cell surface. Actions of insulin: 1. Glucose metabolism – increases glucose uptake by muscles and adipose tissue, suppresses hepatic glucose uptake, increases glycogen synthesis and inhibits glycogen breakdown. It does this by activating the enzyme glycogen synthase and dephosphorylating glycogen phosphorylase kinase.

2. Lipid metabolism – insulin increases the rate of lipogenesis and suppresses lipolysis in several ways in adipose tissue and liver, and controls the formation and storage of triglyceride.

3. Protein metabolism – insulin stimulates the uptake of amino acids into cells and promotes protein synthesis in a range of tissue. 4. Other:  Regulates growth and development and the expression of certain genes  Vasculature – impaired endothelial function, increased stiffness of arteries and procoagulation  Hyperuricaemia – insulin reduces renal uric acid clearance Glucagon • Polypeptide hormone released from alpha cells when there is a fall in BGL (and by amino acids). • Function - increases glucose levels by binding to glucagon receptors in the liver, and accelerating glyconeogenolysis and initiating gluconeogenesis  release of glucose.



Release is stimulated by: Low BGL Exercise + sympathetic division of the ANS Protein meals and amino acids in the blood Release is inhibited by: somatostatin and insulin

o o o •

Diabetes: pathophysiology, diagnosis, symptoms and signs Pathophysiology Type 1 diabetes • Immune-mediated organ-specific disease characterised by an absolute lack of insulin • It is likely that an environmental factor triggers a selective autoimmune destruction of β-cells of the pancreas in a genetically predisposed individual. • Βeta-cells secrete insulin into the blood stream which is essential to the uptake of glucose into body cells therefore glucose builds up in the blood  hyperglycaemia. • The autoimmune nature of type 1 DM is mainly suggested by the presence of T-lymphocytes and macrophages in the islet cells. In addition there are often circulating islet-related autoantibodies such as islet cell autoantibodies, insulin antibodies, glutamic acid decarboxylase autoantibodies. Type 2 diabetes • Relative lack of insulin secondary to two pathophysiological defects: beta-cell dysfunction and impaired glucose tolerance • The combined effect of these is: increased glucose production from the liver owing to inadequate uptake of glucose by skeletal muscle and other peripheral tissues. 1. Impaired insulin action through insulin resistance • Insulin fails to produce its usual biological effects at physiological concentrations. • Causes of insulin resistance:  Absent or ↓insulin receptors – the persistently high insulin levels may suppress the number of receptors present in a process called “down regulation”.  Abnormal insulin receptors – insulin cannot bind, failure of binding to cause activation  Down regulation of postreceptor signalling  Abnormalities of GLUT 4 translocation and function (insulin-responsive transport protein for glucose)  Accumulation of skeletal muscle triglyceride 2. Impaired insulin secretion through a dysfunction of the pancreatic β-cell • Insulin resistance does not account for type 2 DM on its own. • The major beta-cell abnormalities are:  A marked reduction in first-phase insulin secretion  And, in established DM, an attenuated second phase insulin secretion  Usually by diagnosis beta-cell function is <50% with a mean deterioration of 4% per year after diagnosis. • Mechanisms underlying beta-cell dysfunction are likely to be multifactorial:  Early life malnutrition  Hyperglycaemia  Hyperlipidaemia  ??Immunological basis: 10% have islet cell autoantibodies e.g. insulinomas-related antigen (ICA) and GAD (glutamic acid decarboxylase). Risk factors (type 2 diabetes) • Genetics (see genetics section) - unmodifiable • Ethnicity – higher rates in Indians and Africans • Geography – higher in urban populations. Worldwide disease, but highest rates are seen in Pima Indians of Arizona, Europe, N. America. Lowest rates seen in China, Chile. • Obesity - the average BMI of a type 2 diabetic is 30kg/m2 • Fat distribution - ↑visceral fat; “central obesity” • Physical inactivity • Intrauterine environment – low birth weight is a risk factor, babies born to diabetic mothers



Old age - most cases are diagnosed >40y  lifetime risk of 1 in 10. But now type 2 DM is increasingly common in children and young adults.

Genetics of diabetes Type 1 diabetes: • Polygenic inheritance • The greatest contribution to inheritance comes from the histocompatibility (HLA) region on the short arm of chromosome 6, which is important in mounting an immune response to invading organisms. • Examples of high risk HLA genotypes for type 1 DM include: HLA DR3/4 and HLA DQB1. • There is an increased risk of developing type 1 DM if a 1st degree relative already has it – this seems to be further increased if your father as opposed to your mother has the disease – 6% versus 3% • The risk of developing the disease rises to 1 in 2 if you are young and have an affected identical twin Type 2 diabetes: • Polygenic inheritance • The genes responsible are poorly defined, but they are important – the heritability of type 2 is greater than type 1 – it is estimated to account for 40-80% of disease susceptibility. • Twin studies show a high concordance of 80-90% • Approximately 2-5% of cases are caused by single gene mutations such as maturity onset diabetes of the young (MODY). This is an AD condition caused by a mutation in one of six genes, most commonly Hepatic Nuclear Factor 1α (also glucokinase). Clinical presentation Type 1 diabetes: • Age: peak incidence is puberty (6.5%), but it can present at any age • Rapid onset (1-4 weeks) of symptoms in a young patient • Symptoms relating to the osmotic effect of hyperglycaemia o Increased thirst and polydipsia o Polyuria and nocturia o Blurred vision o Drowsiness and dehydration – as water leaves the cells along the osmotic gradient only to be lost into the urine • Cutaneous candidal infection: o Of the vulva  pruritus vulvae o Of the foreskin  balanitis • Symptoms related to lack of “fuel”: o Extreme fatigue o Muscle wasting through protein breakdown o Weight loss • Diabetic ketoacidosis – some patients will present in this way. Diabetic ketoacidosis occurs when the body metabolises fatty acids because there is no glucose available  nausea, vomiting, acidotic breathing, and ketones on the breath (“pear-drops”) Type 2 diabetes: • Acute presentation Young people often present with a 2-6 week history and report the classic triad of symptoms: 1. Polyuria - due to the osmotic diuresis that results when blood glucose levels exceed the renal threshold 2. Thirst - due to the resulting loss of fluid and electrolytes 3. Weight loss - due to fluid depletion and the accelerated breakdown of fat and muscle secondary to insulin deficiency. • Subacute presentation o Insidious onset over several months or years, particularly in older patients. o Thirst, polyuria and weight loss o Lethargy, visual blurring (owing to glucose-induced changes in refraction), or pruritus vulvae or balanitis that is due to candidal infection. • Asymptomatic diabetes

Glycosuria or a raised BGL may be detected on routine examination (e.g. for insurance purposes) in individuals who have no symptoms of ill-health. o Glycosuria is not diagnostic of diabetes but indicates the need for further investigations. About 1% of the population have renal glycosuria. There is an inherited low renal threshold for glucose, transmitted either as a Mendelian dominant or recessive trait. Presentation with complications: o Staphylococcal skin infections o Retinopathy noted during a visit to the optician o Polyneuropathy causing tingling and numbness in the feet o Erectile dysfunction o Arterial disease, resulting in myocardial infarction or peripheral gangrene. o



Diagnosis of diabetes 1. Clinical history i.e. polyuria, polydipsia and unexplained weight 2. PLUS a positive result from one of the following tests: • Random venous plasma glucose concentration of more than or equal to 11.1mmol/l • Fasting plasma glucose concentration of more than or equal to 7.0mmol/l • Oral glucose tolerance test: plasma glucose concentration of more than or equal to 11.1mmol/l 2hrs after 75g anhydrous glucose in 300ml water ingested over 5mins  If the patient is asymptomatic, diagnosis should not be based on a single glucose test – a confirmatory plasma venous determination is required on another day.  Impaired glucose tolerance if plasma glucose concentration was 7.8-11.1mmol/L on OGTT or fasting plasma glucose was 6-6.9 Management of diabetes Lifestyle: diet and exercise • Eat a healthy, balanced diet: o Reduce the amount of refined sugar and fat o Increase the proportion of complex carbohydrates and fibre – slower rate of absorption o Moderate alcohol intake • Lose weight if overweight – adopt a diet with a moderate calorie deficit • Exercise - patients should be encouraged to exercise for 30mins at least per day to improve glycaemic control and reduce their cardiovascular risk • Never smoke Specific management type 1 diabetes • Diet as above • Insulin parenterally – usually SC (sometimes IV or IM) • Avoid vascular complications: o Control blood pressure with antihypertensive regimes and aspirin in high risk patients o Control cholesterol with statins o Check feet and legs for cuts or infection Specific management type 2 diabetes: • Patients follow a “stepwise” approach to management: 1. Lifestyle changes only  diet and exercise – continue for 3m, if the patient continues to be symptomatic proceed to 2. 2. 1 + oral hypoglycaemic agent 3. 1 + combination of oral hypoglycaemic agents 4. 3 + insulin 5. 1+ insulin • Insulin is commenced only if the oral hypoglycaemic drugs fail, or there is intercurrent illness, the patient may be put onto an insulin regime such as: o Basal insulin - this is often a suitable first step, using once or twice daily intermediate- or long-acting insulin

o o •

Twice daily biphasic insulin Basal-bolus regimen Antihypertensives – initiate treatment for hypertension when bp >130/80mm/Hg. Special situations e.g. microalbuminaemia (>135/75) or proteinuria (>125/75).

Monitoring of Diabetes Self-monitoring • Blood Glucose o BGL levels are recommended to be in the range:  Pre-prandial levels of 4-6 mmol/l  Post-prandial levels <10 mmol/l 2 hours after meals o Used by patients who need to adjust insulin levels to their blood glucose and especially during ill health • Urine o It is possible to monitor diabetes in the urine via glucose and ketones. o Glucose:  Glycosuria occurs when BGLs are too high so urine testing of glucose via a test strip (%) can tell you when you have hyperglycaemia.  This method is easy, cheap and painless, but it must not be used in type 1 diabetics or those on sulphonylureas, because urine glucose testing does not detect hypoglycaemia. This method is also affected by fluid intake. o Ketones can also be detected in the urine, but only when you are in a fasting state i.e. severe hypoglycaemia. Ketonuria is a dangerous sign. • How often? o Type 1 diabetic:  If you have good glycaemic control, testing should be carried out once before bed and on one other occasion each day. Vary the timing of the second test so you can build up a profile of your glycaemic control. Some patients prefer to perform frequent tests on 2 or 3 days in the week and then again only if concerned about possible hypos.  Generally test more (qds) if: bad control and recurrent hypos or hypers, ill, using pump/pregnant, nocturnal hypo or resistant hyper (measure also in early morning) o Type 2 diabetic:  Unknown what the ideal number of tests needs to be  If on insulin and oral hypoglycaemic agents – once a day and mix up times of testing so that you can identify trends in your blood sugar  If you control sugars with diet or metformin no need to test because no risk of hypoglycaemia and glycaemic control is adequately monitored by testing glycosylated haemoglobin Measures of long term control • Glycosated Haemoglobin (HbA1c) o Produced by the glycosylation of haemoglobin at a rate proportional to the glucose concentration. It depends on: red cell lifespan and prevailing blood glucose concentration o Providing that the red cell lifespan is normal, HbA1c measures mean blood glucose concentration over the preceding 60 days (half-life of red cell). o Diabetics should aim fro a value of 7.5% or below. • Fructosamine - measure of glycated serum albumin. It is an index of glycaemic control over the previous 2-3 weeks (albumin has a shorter half-life compared to Hb). The Annual Review • All diabetics are reviewed at least once per year by a hospital clinic, a diabetes centre, their GP or a combination of these three. • The purpose of an annual review is: 1. Patient education 2. Optimize targets of therapy e.g. BP 3. Screen for diabetic complications 4. Treat any complications • Lab tests and investigations: o Blood glucose control: check HbA1c

Kidney function: urine/blood tests for protein. Urine protein test - if positive, check microalbuminuria x3. If persists, start ACE inhibitor o Serum creatinine. o Blood fats: lipids, cholesterol and triglycerides. National target ranges are a total cholesterol of below 4.0 mmol/l and a fasting triglyceride of 1.7mmol/l or below. Physical Examination: o Calculate their BMI o Examine legs and feet – check circulation, skin and nerve supply – if necessary refer to chiropodist. o Blood pressure – aim to be 130/80 or below o Fundoscopy to check for retinopathy o If on insulin, check injection sites Lifestyle Issues: o General wellbeing – how the patient is coping with diabetes o Discuss the current treatment – ask about compliance o Discuss the patient’s diabetic control – look at their home monitoring results and discuss their hypos o Discuss smoking, alcohol, stress, sexual problems, physical activity and diet o Discuss any patient concerns o





Pharmacology and diabetes Insulin MOA: (see physiology of pancreas) Types of insulin: 1. Short-acting insulins: (actrapid, novorapid, humalog, velosulin) • Rapid-acting insulin analogues – work in about 15 minutes and peak at 1h, and last ~3-4hs. They can be injected shortly before, during or immediately after meals. These are used in those prone to hypos between meals and avoid the need for snacks between meals because of their faster onset and shorter duration of action. • Soluble insulins – work in about 30 minutes, peak at 2-3h and last for 8h. They should be injected 30 minutes before meals. These are usually given SC, but in hospital they can be given IV via a pump. • Inhaled insulin (Exubera) – this is a fast-acting, dry powder preparation of human insulin that is inhaled before meals, using a specially designed inhalation device. However, only 10% of the dose reaches the circulation and this has cost implications. It is not recommended for routine use in patients but it can be used in a few circumstances e.g. poor BG control despite other treatment options or the pt is unable to inject insulin due to: injection phobia or severe + persistent problems with injection sites. 2. Intermediate-acting insulins: onset of action after 2-4h, peak at 6-7h, and last 20h. Examples: Humulin I, Insulatard 3. Long-acting insulin analogues: onset of action at 1-3h, then plateau, and last 20-24h. They are used once or twice daily, and achieve a steady-state to produce a constant level of insulin. They are used in conjunction with short-acting insulins and provide a “background” insulin dose. Examples include: Insulin glargine (Lantus), Insulin detemir (Levemir), Insulin zinc suspension, Protamine zinc insulin 4. Biphasic insulins: mix of rapid or short-acting insulin with intermediate-acting insulins. Examples: NovoMix 30, Humulin M3, Insuman comb, Humalog Mix25 Insulin regimes: 1. Twice-daily regimen • Biphasic insulin (soluble and intermediate insulins) injected twice a day; 20-30 minutes before breakfast and the evening meal. • The patient must eat regularly at predetermined times and there is little flexibility especially with lunch, because the lunchtime insulin is delivered at breakfast. • High risk of hypos 2. Three-times-daily soluble with intermediate- or long-acting insulin given before meals • This is appropriate for most younger pts • Advantages: the food and the insulin go in at roughly the same time so that meal times and size can vary, without greatly disturbing metabolic control. It is also very flexible so useful for pts with busy jobs, shift workers and those who travel regularly. 3. Basal-bolus regimen

• • •

Intermediate or long-acting insulin is given at bedtime to cover overnight insulin requirements. It is combined with rapid or short-acting insulin injections to cover mealtimes. Offers greater flexibility, and is the most commonly adopted method when intensified insulin therapy is used to provide optimal glycaemic control. 4. Continuous subcut insulin infusion • Indwelling catheter allows an adjustable basal infusion rate of insulin with a small pump strapped around the waist • Indications: useful for those with recurrent hypos, unpredictable lives etc • Advantages: can activate pre-meal boluses, pumps can be disconnected for short periods e.g. swimming, pre-programmed to compensate for nocturnal/early morning glucose changes, rate of insulin absorption more predictable than with multiple SC injections, • Limitations: nuisance of always being attached to a gadget, skin infections, ketoacidosis risk if the flow of insulin is broken and cost. Complications of insulin • Allergy (local or generalized): circulation insulin antibodies can affect insulin action • Insulin resistance – the most common cause of mild insulin resistance is obesity. Insulin resistance associated with antibodies directed against the insulin receptor has been reported in pts with acanthosis nigricans. • Dose dependent effects: hypoglycaemia and weight gain (insulin also increases appetite) • Dose-independent effects - lipohypertrophy caused by repeated injections at the same site, lipoatrophy (rare) and insulin oedema • Insulin during illness  maintain insulin (or even increase dose) because usually an illness will increase BGL especially if the pt has a fever Oral hypoglycaemic agents 1. Insulin secretagogues  Stimulate insulin release from the pancreas.  Sulphonylureas and Meglitinides 2. Insulin sensitisers  These drugs have no effect on insulin secretion but they can improve the effectiveness of insulin already present in the circulation.  Biguanides (metformin) and glitazones 3. Decrease absorption of glucose from the gut  Acarbose Sulphonylureas • Examples: glibenclamide, glipizide, glicazide, tolbutamide • MOA: increase insulin release from beta-cells. The drug binds to a sulphonylurea receptor leading to closure of the K-ATP channel and thus a ↑ in intracellular calcium and in turn the release of insulin. • S/Es: o Weight gain – patients will put on a lot of weight particularly because previously when their diabetes was poorly controlled, they lost energy through glycosuria. o Hypoglycaemia – 2% of patients suffer this side effect. The drugs interfere with normal glucose homeostasis. o Hyponatraemia – the drug increases the sensitivity of the DCT to ADH. o Worsening of MI (?) – potassium channels also exist in cardiac myocytes o Bone marrow damage is a severe but very rare complication • Pharmacokinetics: taken orally, reach peak plasma concentrations within 2-4hours. Most are secreted in the urine so caution must be taken in anyone with renal impairment particularly the elderly because of the risk of severe hypoglycaemia. They should not be used in pregnancy because they can cross the placenta. • Drug interactions: NSAIDs, coumarins, alcohol, monoamine oxidase inhibitors and some antibacterial drugs (sulfonamides, trimethoprim) – if combined they can produce severe hypoglycaemia Meglitinides • Examples: nateglinide and repaglinide. • MOA: stimulate insulin release by closing the K-ATP channel – they bind to a different receptor to the sulphonylureas. They are designed to restore the early phase post-prandial release of insulin without prolonged stimulation during periods of fasting.



S/Es: hypoglycaemia (less common than with sulphonylureas because of their short duration of action).

Metformin • MOA: lowers BGL by complex mechanisms not yet understood o Increases glucose uptake by skeletal muscle and adipocytes o Suppresses hepatic gluconeogenesis o Reduces glucose absorption from the small intestine o May stimulate AMP kinase, an enzyme which activates glucose transporters and facilitates its uptake into cells. May also suppress appetite and thus help achieve weight loss. • Clinical use: metformin is the drug of choice in treating type 2 diabetics. Metformin can be combined with sulphonylureas, Glitazones or insulin. Should be taken with or after food. • S/Es: GI disturbances including: anorexia, diarrhoea and nausea (these can usually be avoided if the dose is gradually increased). A fatal but rare complication is lactic acidosis because it is associated with an increase in lactate production by inhibiting pyruvate metabolism – therefore when there is impaired clearance of lactate or an increase in anaerobic metabolism such as shock, lactic acidosis can result. • Pharmacokinetics: prevents hyperglycaemia but it does not cause hypoglycaemia. It reduces cardiovascular risk by reducing LDL and VLDL cholesterol. It has a half life of 3hrs and is excreted unchanged in the urine. Thiazolidinediones (glitazones) • Examples: Rosiglitazone and Pioglitazone • MOA: bind to a receptor in the cell nucleus called peroxisome proliferator-activated receptor-gamma (PPARγ). This receptor is abundant in adipose tissue and this is the major site of action of these drugs but it is also found in muscle and liver cells. They reduce hepatic glucose output and increase glucose uptake into muscle, which enhances the effectiveness of the insulin produced by the body. This reduces the amount of insulin needed to maintain a given level of blood glucose by about 30%. • S/Es: weight gain on hips and thighs, fluid retention which can precipitate any cardiac failure, dilutional anaemia. An old glitazone called troglitazone was associated with liver toxicity so regular LFTs are recommended. • Pharmacokinetic effects  they can increase sodium reabsorption in the kidney  fluid retention. They may take up to 3 months to reach their maximal effect because their effect on BGL is indirect. Also increase HDL cholesterol  can reduce cardiovascular disease risk. Acarbose • MOA: competitive inhibitor of α-glucosidase in the brush border of the small intestine which hydrolyses disaccharides here  reduces glucose uptake from the gut. The post-prandial peak of blood glucose is reduced and so BGC is much more stable throughout the day. • Indications: type 2 diabetics who cannot control their diabetes with diet alone • S/Es: GI disturbances particularly flatulence, abdominal distension and diarrhoea as unabsorbed carbohydrate is fermented in the bowel. Microvascular complications Pathogenesis • Multifactorial • Hyperglycaemia: o Advanced glycation end-products (AGE) – prolonged exposure of proteins to high glucose levels causes glucose binding and glycation products. Initially this is reversible, but eventually there is irreversible cross-linking and development of AGEs. They accumulate in proportion to hyperglycaemia and time. They promote extracellular matrix formation which impairs the function of tissues. Antioxidants may impair AGE formation. o Reactive oxygen species – are increased leading to reduced nitrous oxide, with loss of its antiinflammatory, antiproliferative and anti-adhesive properties. o Activation of NFκB – this is an intracellular transcription factor that mediates pro-inflammatory responses o Sorbitol pathway – excess glucose is metabolised to sorbitol via the polyol pathway by aldose reductase. This pathway uses NADPH and NAD+ eventually decreasing levels of reduced glutathione which is important in getting rid of reactive oxygen species that are damaging to cells.

o Activation of protein kinase Cβ – When glucose is metabolised the expression of PKCβ is increased and





this in turn increases the expression of a number of mitogenic cytokines such as transforming growth factor β and vascular endothelial growth factor. o Cytokines – AGE products also increase levels of the afore mentioned cytokines  ↑vascular permeability and angiogenesis – this may be responsible for macular oedema and new vessel formation seen. Haemodynamic theory of diabetic complications – proposes that microvascular complications result from chronic abnormalities in the blood flow through capillary beds which is thought to be brought on by hyperglycaemia. The osmotic effect of hyperglycaemia  high flow rates that are increased further in patients with hypertension  tissue damage due to impaired nutrient and oxygen supply. The growth hormone-insulin-like growth factor axis – there is reduced hepatic production of insulin growth factor in diabetics often in response to artificially administering insulin and hence the body “forgets”. This leads to reduced feedback to the pituitary gland and growth hormone hypersecretion (2-3x). Growth hormone stimulates glucose release and has a diabetogenic effect.

Diabetic retinopathy Pathogenesis: • Hyperglycaemia – as above • Glycosylation of tissue proteins may play a major role • Loss of the blood-retinal barrier - twin processes of small vessel occlusion and increased permeability – there are changes to the vessel wall and loss of supporting pericytes • Ischaemic tendency – caused by an increased red cell and platelet stickiness and reduced oxygen transport. In response to the ischaemia the retina releases vasogenic factors which cause proliferation of new vessels. Natural history: 1. Background of retinopathy  dot haemorrhages (microaneurysms), blot haemorrhages (small intraretinal haemorrhages) or hard exudates (lipid or protein exudates) 2. Pre-proliferative retinopathy  cotton wool spots that result from retinal ischaemia and/or intraretinal microvascular abnormalities (clusters of irregular branched vessels in the retina and may represent early new vessel formation), venous beading/loops. 3. Proliferative retinopathy  new vessel formation due to growth factor release in response to retinal ischaemia/hypoxia. The vessels are weak and have a tendency to bleed, which can lead to blindness because there can be a fibrous tissue reaction. There can be a vitreous haemorrhage, which involves loss of vision in one eye, on waking, or as a floating shadow. It is seen as a featureless, grey haze with ophthalmoscopy. Partial recovery of vision occurs as blood is absorbed but repeated bleeds may occur. 4. Advanced retinopathy  retinal fibrosis, traction retinal detachment, loss of vision, retinal detachment. 5. Maculopathy  retinopathy occurring around the macula  reduction in visual acuity Management: • Screening – annual eye test of visual acuity and fundoscopy with retinal photography and fluorescein angiography. Start screening: type 1 DM within 3-5 years of diagnosis after age 10y; type 2 DM at the time of diagnosis; screen all pregnant women prior to conception and during the 1st trimester if they have DM. • Prevention: good glycaemic control (a 2% reduction in HbA1c can halve the incidence and progression of retinopathy), effective control of hypertension, stop smoking • Treatments:  No drugs are licensed however, protein kinase C inhibitors may have the potential to improve retinopathy  Laser photocoagulation - destroys peripheral parts of the retina thus reducing the ischaemic stimulus for new vessel formation. This sacrifices peripheral vision for central vision but it is effective.  Vitrectomy may be required if an intravitreal haemorrhage fails to clear. Eye disease and prognosis: • Retinopathy develops insidiously and almost always asymptomatic until patient has a catastrophic intraocular sight-threatening haemorrhage • Other eye problems seen in diabetes: cataract (2ndry to osmotic changes in the lens), refractory defects (2ndry to osmotic pressure changes in lens), glaucoma, infections, nerve palsies • Statistics:  Most common cause of new cases of blindness among adults 20-74yrs

 12000-24000 people lose their sight due to DM each yr  Affects 1 in 3 diabetics  5% are blind after 30yrs of diabetes Diabetic nephropathy Pathogenesis • Ischaemia - resulting from hypertrophy of afferent and efferent arterioles • Ascending infection – urinary infections are more common in diabetics. This is because of: urinary stasis due to autonomic neuropathy affecting bladder function, high glucose levels of urine. • Glomerular damage and renal failure 1. Functional changes  Increase in GFR – secondary to polyuria (greater urine volumes)  Expansion of tubular tissue and enlargement of kidneys – secondary to ↑renal filtration. This leads to glomerular hypertrophy and oxidant stress. The result is premature glomerulosclerosis. 2. Structural changes  AGE products cause irreversible protein cross-linking in the glomerulus and basement membrane  Disruption of protein cross links and basement membrane thickening occur  There is also dilation of the afferent arteriole more than the efferent arteriole leading to a rise in intraglomerular filtration pressure, further damaging the glomerular capillaries. 3. Microalbuminuria  Disruption of protein cross-links alters the glomerular filter allowing progressive leak of large molecules into the urine  There is microalbuminuria and in 20% progression to proteinuria and nephrotic syndrome  The GFR decreases and serum creatinine rises 4. Overt clinical nephropathy - as glomerular filtration fails so the blood pressure and plasma creatinine rise and proteinuria increases. 5. End-stage renal failure – anaemia, altered calcium metabolism (↓Ca and ↑phosphate), dyslipidaemia, and hypertension. Prevention of nephropathy • Essential as diabetes is the main cause of renal failure in Europe. Clinical nephropathy appears between 15-25y after diagnosis. • Prevention: o Optimise glycaemic control (HbA1c <7%) – metformin should not be used with a raised creatinine, insulin therapy should be initiated when it goes >200. o Normalisation of bp (<130/80mm/Hg) + correction of CV risk factors – treatment with ACE inhibitors (e.g. –pril) is the optimal drug even in normotensive patients if they have microalbuminuria. Other antihypertensives used include angiotensin receptor blockers (ARBs) such as candesartan (which should be used if there is an intolerance to ACEs), or B-blockers. o Other therapy: restrict dietary protein to RDA of 0.8 g/kg body weight per day; stop smoking; reduce lipids. • Screening – via urine dipstick for proteinuria, 24-hr urine sampling or albumin:creatinine ratio. For confirmation of microalbuminuria, 3 urine tests should be positive and other causes of proteinuria excluded. • When to refer - when to refer microalbuminuria is determined by a locally defined protocol – referral to a nephrologist should be considered with increasing proteinuria, ↓GFR <60mL/min and ↑serum creatinine to 200-250umol/L. Diabetic neuropathy Pathophysiology • Occlusion of vasa nervorum (blood vessels supplying nerves) secondary to atherosclerosis • Hyperglycaemia – slows nerve conduction and causes uncomfortable sensory symptoms. It also causes an increase in the formation of sorbitol and fructose in schwann cells – so that an accumulation of these can disrupt function and structure  segmental demyelination Patterns of diabetic neuropathy:

1. Acute sensory neuropathies • •

Usually asymmetrical mononeuropathies which are transient. Diffuse and painful – burning or crawling pains especially at night. This neuropathy usually resolves after 3 months • Focal mononeuritis and mononeuritis multiplex – any nerve can be affected causing radiculopathies, isolated cranial nerve palsies, carpel tunnel etc. Onset is rapid but there is always a full spontaneous recovery. 2. Chronic, symmetrical, sensory polyneuropathy • Very common • There is loss of vibration sense, pain sensation, and temperature sensation in the feet and hands mainly – glove and stocking distribution • At later stages there can be balance problems and impaired proprioception (“walking on cotton wool”). • Complications include: trauma at pressure points, blistering, clawed toes, callus formation, ulcers, and eventually neuropathic arthropathy i.e. Charcot’s joints. 3. Acute motor neuropathies • This is rare and can lead to amyotrophy or muscle wasting. • Quite prevalent in old men with diabetes whereby they have weight loss and painful, asymmetrical muscle wasting, usually of the quads. The wasting may be so marked that there is no knee reflex and the area affected becomes quite tender. • Amyotrophy is usually associated with poor glycaemic control and will resolve if control is improved. 4. Autonomic neuropathy • Affect the sympathetic and/or parasympathetic nervous systems • CV system  vagal neuropathy  tachycardia at rest and loss of sinus rhythm; postural hypotension; peripheral vasodilatation. • GIT  gastroparesis due to vagal damage (can cause vomiting); diarrhoea • Bladder  loss of tone  incomplete emptying  stasis  ↑infections • Erectile dysfunction Management • Control blood pressure • Introduce insulin • Pain-killers such as amitriptyline, carbamazepine, gabapentin etc • Where there is sensory loss in feet: immobilisation, custom-made footwear, reconstruction, IV bisphosphonates. Macrovascular complications Pathogenesis - atherosclerosis 1. Chronic endothelial injury: • Due to physical forces e.g. turbulent flow at points of bifurcation • Due to toxins e.g. cigarette smoke, hyperlipidaemia, hyperglycaemia, viruses, immune reaction 2. Endothelial dysfunction: • Increased permeability • Increased expression of cell surface adhesion molecules that recruit leukocytes • Altered release of vasoactive substances • Release of inflammatory cytokines • Impaired production of antithrombotic agents (nitrous oxide or antioxidants) which are promoted by a laminar flow 3. Lipoprotein entry and modification: • The Dysfunctioning endothelium allows circulating lipoproteins to pass through and into the internal elastic intima particularly in areas of haemodynamic strain • LDL cholesterol accumulates in the subendothelial space and binds to proteoglycans  fatty streak • The lipids can be oxidised and altered by enzymatic change or even glycated (in diabetics) to form inflammatory lipids that attract inflammatory cells 4. Leukocyte recruitment: • Inflammatory, antigenic lipids and the release of inflammatory mediators by the Dysfunctioning endothelium, encourage monocytes and T-lymphocyte entry



Monocytes:  Mature into macrophages on entry  Produce: IL-1, TNF-alpha and MCP-1 increasing adhesion and recruitment of leukocytes to plaque  Engulf oxidised LDL to form foam cells and present the oxidised LDL as an antigen to T-lymphocytes causing their activation • T-lymphocytes:  Produce inflammatory cytokines, INF-gamma, lymphotoxin, IL-1, TNF-alpha setting up the chronic inflammatory state  Stimulate macrophages via oxidised LDL as well  Try to heal the blood vessel by producing ground substance (collagen) and fibrous tissue, but this only thickens the vessel and helps in plaque formation 5. Plaque formation: • Macrophages, T-lymphocytes and platelets release growth factors (PDGF, FGF, TGF-a) which cause smooth muscle cell migration from the arterial media to the intima • Subsequent smooth muscle proliferation, extracellular matrix formation, fibrous tissue release and foam cells modified with inflammatory mediators form an atheromatous plaque. • The plaque has a thrombogenic lipid core (tissue factor-concentrated) and a protective fibrous cap 6. Plaque instability and rupture: • The continuous cycle of matrix synthesis and degradation by inflammatory cytokines with cell necrosis and release of contents, the lipid core grows • The plaque may fissure and release its thrombogenic core  STEMI or NSTEMI The diabetic foot Pathophysiology – combination of neuropathy, ischaemia and infection Charcot’s arthropathy 1. Acute onset – acutely swollen, hot foot. Only a 1/3 have pain. The initiating event is often a trivial injury. Differential: gout, cellulitis, DVT. At this point, the foot should be immobilised in a non-weight-bearing cast. 2. Bony destruction – if treatment of the acute stage is delayed then the foot can become deformed as bone is destroyed. These changes can happen very quickly, in just a matter of weeks. Any deformity predisposes the foot to ulceration, particularly on the plantar surface. Immobilisation is essential. 3. Radiological consolidation and stabilisation – the destructive process stabilises after 6-12m. Arterial leg ulcers • Painful, small, punctuate ulcers that are usually well circumscribed and usually on the dorsum of the foot • Pulselessness, pallor, cool skin • Delayed capillary return time • Atrophic appearing skin (shiny, thin, dry) with loss of digital and pedal hair Acute ischaemia • Pain - sudden onset, continuous, variable intensity; often at the junction of perfused and ischaemic tissue, usually in one periphery. • Paraesthesia - sudden onset, usually in one periphery; may be an altered or a complete absence of sensation. • Pallor - periphery is white and may become blue with onset of necrosis. There is poor peripheral capillary return on pressure blanching and the skin may blister. Buerger's test is positive. • Perishing cold • Pulselessness - foot pulses are absent, and may be undetectable by Doppler ultrasound. Popliteal and femoral pulses may be absent depending on the level of occlusion. • Paralysis - indicates extreme ischaemia. The movements of flexion and extension of the ankle and toes are eventually lost. Chronic ischaemia • Mild ischaemia presents with intermittent claudication • Critical ischaemia: rest pain, poor healing, ulcers and pressure sores Investigations of the diabetic foot: ischaemia • Listen for arterial bruits with a stethoscope of the artery (you might hear a whooshing sound) • Decreased or absent pulse in the extremities • Decreased blood pressure in affected limb

• • • •

• •

High cholesterol Claudication distance inquiry and exercise testing – normally foot pressure should increase with exercise but it drops in the case of occlusive arterial disease (the size of the drop gives you an indication of severity). Ankle/Brachial Index (ABI) <1; severe ischaemia if <0.4 Doppler ultrasound examination of an extremity – this examines blood flow in the major arteries and veins of the arms and legs. You can use a Doppler machine to amplify the korotoff sounds when measuring blood pressure in the ankle. Angiography of the arteries in the legs Distinguish between an ischaemic and a neuropathic foot: Symptoms

Ischaemia Claudication Rest pain

Inspection

Dependent rubor Trophic changes

Palpation

Cold Pulseless Painful heals and toes

ulceration

Neuropathy Usually painless Sometimes painful neuropathy High arch Clawing of toes No trophic changes Warm Bounding pulse Painless Plantar

Treatment of the diabetic foot Foot ulcers: • Prevention: carefully measured footwear, wash feet daily + dry well, cut nails carefully + regularly, inspect feet daily, use moisturising creams for callus or fissures. • Treatment: o Relief of pressure by bed rest or contact casting o Radical debridement o Treat any infection with appropriate antibiotics – common infectious agents are strep pyogenes and staph aureus. Antibiotics should be broad-based and given for 1m. Antibiotics used: penicillin-cloxacillin combination, metronidazole (anaerobes), cephalosporin, vancomycin (for resistant bacteria). For severe infection, consider osteomyelitis. o It may be necessary to perform revascularisation of ischaemic limbs. o Amputation – last resort because it can result in w/c use, and can ↑risk of ulceration. Peripheral artery disease: • Self care is essential – it is good to walk to the point of pain alternated with rest periods because it allows new, smaller blood vessels to develop. Stop smoking! Good foot care! Low fat diet! • Medication: analgesics, anticoagulants, antiplatelets such as aspirin 75mg od, vasodilators (Naftidrofuryl 100200mg tds daily or Cilostazol 100mg bd) and a statin • Surgery – this is usually only performed in the most severe cases. Examples of surgery includes: removing the lining of the artery (endarterectomy), repair/replacement of a vessel (grafting), bypass using a vein or synthetic graft. • Alternatives to surgery are treatments such as balloon angioplasty, stent implantation

Surgery: • Angioplasty – a catheter is introduced percutaneously into an artery and a balloon is inflated inside an artery to attempt to widen a narrowing. Patency is about 95% for 2 years for lesions above the inguinal ligament or 80% below. Complications include: rupture of the vessel, embolism, or damage at the entry site for angioplasty. • Stenting –a cage-like stainless steel support device is inserted collapsed into the right place in the blood vessel and then expanded into the open position by inflating a high pressure balloon in its interior. The stent will act as a scaffold for the artery and maintains blood flow. The stent is thrombogenic however, and so a combination of oral antiplatelet agents e.g. aspirin + clopidogrel, is crucial after stent implantation. Stenosis rates are much lower with stents, but there is still a problem due to migration of smooth muscle cells. Drug-eluting stents have

• • • •

been devised to reduce re-stenosis. These stents are fabricated with a polymer coat that incorporates an antiproliferative medication. Thrombectomy – excision of a thrombus Thrombolysis – use of thrombolytic drugs to dissolve a thrombus Endarterectomy – surgical removal of an atherosclerotic plaque. Bypass grafting – a good vein is attached to the area of atherosclerosis and acts as a bypass. Usually this is the greater saphenous vein. The vein must be inverted so that the valves do not close and block off the blood flow or the valves can be stripped off; depends on the technique preferred. If there are no suitable veins, then a synthetic tube can be used – this always has a higher complication rate. Any sort of bypass can be performed e.g. iliofemoral, fem-fem, femoropopliteal, femorodistal/femorotibial etc.

Erectile dysfunction • Prevalence increases with age – 60% of men with DM >60y are affected. Women may also suffer sexual dysfunction - there is an increased risk of vaginal dryness and impaired sexual arousal and genitourinary infections, in particular candidiasis, are common in diabetic women. • Pathophysiology - penile erection occurs following the flow of blood into the erectile tissue. Nitrous oxidemediated vascular smooth muscle relaxation of the corpus cavernosum produces expansion of the cavernosal space and compression of the outflow venules. This allows blood to flow into, but not out of the penis. Erectile dysfunction in DM mainly results from autonomic neuropathy and endothelial dysfunction. Other factors can also contribute: drugs, psychological, neurological, endocrine, and metabolic disorders (hypothyroidism). • Treatment: o Phosphodiesterase type-5 inhibitors – e.g. sildenafil, tadalafil, vardenafil. These inhibit the breakdown of cyclic GMP, which is a second messenger of NO. So they enhance the effects of NO on smooth muscle and increase penile flow. These drugs should be taken before intended sexual activity because they enhance erections under conditions of sexual stimulation. They are effective in ~50-60%. Patients must take care not to use these drugs with nitrates because they may cause severe, acute hypotension. o Prostaglandin E (alprostadil) - Administered by injection into the corpus cavernosum or transurethrally o Apomorphine - 2 or 3 mg sublingually 20 minutes before sexual activity. It is a dopamine agonist. o Vacuum devices - draw blood into the penis while a constriction band around the base of the penis prevents blood leaving the penis and thereby maintains the erection. o Surgery - is a last resort. It involves the insertion of a plastic rod in to the penis so that penetration can be achieved. Prevention of macrovascular complications Lifestyle measures: • Quit smoking – reduces risk of progression to CVD by up to 70% in the non-diabetic population • Increase physical activity • Loss weight • Balanced, healthy diet • Clean and check feet regularly - ? chiropodist Control BGL • For every 1% fall in HbA1c there is a reduction in microvascular risk by about 25% irrespective of whether the patient has type 1 or type 2 DM. • Glitazones/Thiazolidinediones are effective in controlling multiple risk factors AND BGL Control blood pressure • When bp >130/80mm/Hg, the risk of complications increases, so current proposals aims for these levels. • ACE inhibitors are particularly good because of their role in renal protection - lowers the risk of a heart attack, stroke, overt nephropathy or CV death by 25-35%. Control of blood cholesterol • Lipid-lowering therapy: o Statins – simvastatin produced a 55% reduction in the incidence of major CHD. Pravastatin also good. o Fibrates or nicotinic acid are helpful in patients with low HDL cholesterol despite statin therapy Use of aspirin • Patients with a risk of CV events of >20% in the next 10y have a reduced mortality with aspirin • If intolerant, clopidogrel is a good alternative • The benefits outweigh the bleeding risk when: o Age 45 with 3 strong CV risk factors o Age 45-54 with 3 strong CV risk factors

o

>65 with 1 CV risk factor Psychosocial and Economics of Diabetes

Social implications 1. Work • Risk of hypoglycaemia from insulin therapy may put others at risk • Can apply for registration as a disabled person, which can help in finding a job • Shift work, long working days, international travel can all impact insulin regimens. Meals can be early or late, sometimes meals are missed, and sometimes there are frequent business lunches. • Generally excluded from: o Vocational driving: LGVs, PCVs, taxi drivers, chauffeurs, underground trains o Civil aviation: commercial pilots, flight engineers, aircrew, air-traffic controllers o National and emergency services: armed forces, police force, fire brigade or rescue services, prison and security services o Dangerous areas for work: offshore oil-rig work, moving machinery etc o Work at heights: crane driving, scaffolding etc 2. Finance • Life and car insurance need to be informed if diabetic and on insulin. Premiums are often dependent on the risk of hypos and the presence of complications. • In UK they are exempt from prescription charges (GP signs a SP92 form) • Small, supplementary pensions are available to help diabetics with the increased cost of food for their diets 3. Sport • Can play most sports • Wary in: scuba diving, motor rally, boxing • Should avoid dangerous situations e.g. swimming alone • Intense exercise warrants better control: measure BGL before exercise, take 20g carbohydrate every 45mins, keep fast-acting glucose preparations in pocket. 4. Holidays and travel • Can be very difficult; extra precautions need to be taken (medic Alert bracelet, letter to confirm they can carry syringes on aeroplanes) • Time differences make insulin dosing difficult: ?supplementary injections • On day of travel, aim for higher BGL + take regular BGL checks • Sea sickness – vomiting – continue insulin • Looking after insulin: topical climates may require refrigeration, do not leave it in luggage hold as it may freeze • Soluble insulin is usually available in most countries 5. Driving • All diabetic otherwise fit and well can hold ordinary driving licence but the law demands that all patients whether type I or II should inform the DVLA of their condition. o If on insulin licenses are restricted to 1,2 or 3 years only o Patients are only refused a license if erratic control, hypoglycaemia or poor eye sight • Visual acuity and fields must be assessed to determine suitability for driving. • The DVLA require patients to sign a declaration allowing their doctor to disclose medical information about them if they are treated with insulin. • Patients must plan their journey, not drive for >2h, always take glucose. If they experience warning symptoms of hypoglycaemia they should stop, switch off the engine and leave the car, since otherwise they may be charge of driving under the influence of drugs. 6. Surgery • ↑risk of post-operative death • Metabolic control should be optimised before the operation • Generally stop all tablets before surgery and consider insulin treatment (if tablet-controlled) • If insulin-controlled  Substitute short-acting insulin the day before surgery and perioperatively, use IV 10% dextrose with potassium chloride. Post-op, maintain this infusion, until the patient can eat. Psychological implications • Low self image, loss of self-esteem, denial

• • • • • • • •

• •

Social isolation - adolescents can feel isolated and they don’t want to be different from their peers, therefore often going out and drinking large quantities of alcohol, despite knowing the risks as a diabetic. Much anxiety about future: driving, fertility Eating disorders are more common in diabetics – 30-40% of young women will have a significant eating disorder Serious psychological distress is 90% higher in adults with DM - many experience episodes of not coping, of helplessness. Psychiatric problems are 3x more common in adolescents: the difficulty to cope is magnified and there is often a period of poor metabolic control and then later re-emerging with complications. Diabetics often do not get much sympathy or concessions as it is a the presence of the disease is hidden Treatment is complex and demanding. Diabetics may feel uncomfortable/embarrassed/different having to inject insulin in public places. The use of insulin pens and meters mirroring the shape of a pen or MP3 players might help to reduce embarrassment and to cope with the alteration of image. Embarrassing loss of control over personal behaviour Risk taking behaviour is effected e.g. alcohol consumption, unplanned pregnancies, tobacco

Economic impact of diabetes on NHS • 10% of the total UK health budget • Costs the US nearly $132 billion a year – 92 billion in direct medical costs and 40 billion in indirect costs due to lost productivity. • The first Wanless report (2002) estimated the total annual cost of diabetes to the NHS to be £1.3 billion. • Average yearly health cost for a person with diabetes was $13,243 in 2002, compared with $2,560 for someone without diabetes • Life expectancy is decreased by 5-10 years Patient education and diabetes • NICE states 3 goals of patient education: 1. Control of vascular factors e.g. blood glucose, blood lipids, blood pressure 2. Management of diabetes associated complications, if and when they develop 3. Maintaining good quality of life • NICE guidelines on how to educate: o Delivered by an appropriately trained and skilled MDT (at least a Specialist Diabetes Nurse + Dietician) o Group work is less costly and a more efficient use of health professionals’ time, as well as allowing patients to support one another. o Accessible to all  hold meetings in a local community centre? o Use a range of educational methods to “promote active learning” • Special patient education projects: 1. DAFNE (Dose Adjustment For Normal Eating) o Designed for Diabetes Type I patients so that there insulin dose is adjusted according to carbohydrate and activity levels. o ↑Freedom with meals, independence o Patients have a small booklet which lists every food type and its carb count so that they can adjust the insulin dose respectively. o It is an outpatient scheme, delivered over 5 days to groups of between 6 and 8 patients. It costs £545 to educate a patient via DAFNE. 2. DESMOND (Diabetes Education and Self Management for Ongoing and Newly Diagnosed) o Designed for Diabetes Type II patients o It aims to help patients identify their risk factors and possible diabetic complications. o Still quite new Economics of Chronic Disease Economics of chronic disease • What is chronic disease? – a non-communicable life-long or long-term disease such as asthma, RA, cancer, COPD, diabetes and heart failure, that can be controlled but not cured.





The prevalence of chronic disease is increasing – approx 17.5 million adults (1/4 of the UK population) suffer from a chronic condition, an average of 6 in 10 adults. By 2030 it is estimated that the incidence of chronic disease in the over-65s will more than double. Reasons for the increasing prevalence:  Reduction in mortality at younger ages  Demographic Transition – longer life-expectancy (proportion elderly aged 80+ increased from 17% to 24%), therefore more people and more chronic disease; but also with old age comes degenerative and acquired chronic diseases  Health transition – great lifestyle risks – diet, exercise, smoking, alcohol trends

The costs of chronic disease: • The World Health Organisation has identified that chronic conditions will be the leading cause of disability by 2020 and that, if not successfully managed, will become the most expensive problem for health care systems. • Direct costs o Medical: drugs prescribed, staff time  80% of GP consultations, 60% of hospital beds, 60% of emergency admissions have exacerbation of chronic condition, Pts with >1 chronic disease cost 6x as much, Pts with >1 chronic disease make higher use of health care (eg. >3 diseases; use 30% bed days) o Patient: transport; out-of pocket expenses. • Indirect costs: production losses; there are often problems with staying in work • Intangible costs: pain, suffering, adverse effects, lifetime costs (dialysis) What is the solution to the rising costs of chronic disease? 1. Prevention of the disease 2. Self-care and self-management  Support people to take an active role in managing their own care  Help people to manage their specific conditions and to adopt approaches that prevent these conditions from getting worse and reduce the risk of getting further conditions  E.g. expert patient programmes (see below) 3. Disease management and treatment – multidisciplinary teams providing high-quality, evidence-based care, including the use of pathways and protocols 4. Case management – active management of high-risk people with complex needs 5. Knowledge management – identify at-risk groups within the population, carry out needs assessments, understand resource and activity levels and identify trends. Expert patient programmes • Piloted between 2002 and 2004; now part of the NHS Improvement Plan • Lay-led, group-based support for people with chronic diseases to help them in the self-management of their long-term condition. Helps patients take control over their disease and lives. • The programme is delivered locally by a network of trainers and volunteer tutors with long-term conditions. • Relies on the fact that patients know their condition best and know how to manage it, but they need support in this • The programme focuses on five core self-management skills: 1. Problem solving 2. Decision making 3. Resource utilisation 4. Developing effective partnerships with healthcare providers 5. Taking action – develops confidence and motivation, plans for the future • Benefits of self-management include: o Reduced severity of symptoms o Significant decrease in pain o Improved life control and activity o Improved resourcefulness and life satisfaction  Overall improved QoL and reduced incapacity! • Reduced burden on NHS as aim of these programs is to cease involvement with healthcare systems and they also improve outcomes: o 50% reduction in unplanned admissions o 50% reduction in bed-days

o

Significant education in medications compliance

o However: moderate increases in clinic visits, test ordering and consultations







Economics and epidemiology of obesity and links with type 2DM Metabolic syndrome (Syndrome X) Increasing problem in developed world – UK, 50% of adults are overweight ( BMI 25-29.9), 20% are obese (BMI ≥ 30) – USA, 66% of all adults are currently overweight, 33% are obese – Fat deposition results from discrepancy between energy consumption and expenditure Specific causes (a few cases of obesity) • Endocrine factors (Hypothyroidism, Cushing’s syndrome, Hypopthalamic tumours, Insulinoma) • Drug Treatments (Tricyclic antidepressants, sulphonylurea drugs, corticosteroids, some oral contraceptive pills) • Genetic (Prader-Willi syndrome – childhood obesity with abnormal appearance and CNS function) In general though obesity arises from a complex interplay of behavioural and genetic factors

Fitness rather than fatness is the problem

“58%of Diabetes Cases Globally Can Be Attributed to Body Mass Index Above 21 Kg/m2”

Estimated total direct healthcare costs of diabetes in selected European countries Country General healthcare cost Additional cost due to Annual cost per patient with per person (US$) presence of diabetes (US$) Type 2 diabetes (US$) Belgium 1,495 1,647 3,142 France 1,979 1,009 2,988 Italy 1,259 1,611 2,870 Sweden 1,710 855 2,565 UK 1,144 811 2,026 Criteria for Undertaking Community-based Interventions Common and serious disease. Strong causal relationships between risk factor levels and disease risk. Predominantly social factors which determine risk levels e.g. lifestyle behaviours. Established benefit and safety of interventions. Potential for community control exists. Added value to “community” based rather than individual based approach. Study looking at weight loss with 3 interventions

Trial stopped 1 year early, after 2.8 yr of follow-up after as it was to be unethical not to offer everyone lifestyle advice. Effect of interventions on prevalence of diabetes in at risk (obese) groups 3 year data Risk reduction 29% Diabetes in controls 58% whole group 14% in Diet and exercise 71% those aged >60yrs 22% in Metformin 31% Metformin (less effect in older and less obese) Screening for overweight people: takes place as a result of certain government targets e.g. for diabetes, but could be more widely incorporated. Solutions: • Structured programme geared towards prevention • Commitment of more resources- Costs need to be considered as investment • More Education to the users especially the socially deprived • Implementation of the intensive programmes of management to curtail the costs. Costs of obesity: health

The House of Commons Health Select Committee (HSC) estimates that the total cost of obesity [i.e. for those with a BMI greater than 30] and its consequences in England in 2002 was around £3340–3724 million. If the costs of being overweight (BMI 25–30) are also taken into account, the HSC speculatively suggests (assuming costs to be half that for the obese) that the total annual cost of obesity and overweight would be around

£6.6–7.4 billion. Of this total, around £991–1124 million relates to the direct healthcare costs of treating obesity and its consequences, comprising general practitioner consultations, inpatient and day case admissions, out-patient attendances and drug costs. This equates to 2.3–2.6% of total net National Health Service (NHS) expenditure in 2001/02. The vast majority of this total was attributable to treating the consequences of obesity (including cardiovascular disease, type 2 diabetes, stroke, angina, osteoporosis and various cancers) rather than treating obesity itself. Costs of obesity: employment

Lost earnings (lost potential national output) directly attributable to obesity were estimated to be £2350– 2600 million. Of this, around £1050–1150 million was due to lost earnings as a result of premature mortality attributable to obesity. Around 34 000 deaths annually are attributable to obesity, one-third of which occur before retirement age. These account for an annual total of 45 000 lost working years. The remaining £1300–1450 million was accounted for by lost earnings as a consequence of certified sickness. There were around 15.5–16 million days of certified incapacity directly attributable to obesity in 2002. These costs do not include costs associated with uncertified sickness absence. Metabolic Syndrome Metabolic syndrome is a common condition also known as dysmetabolic syndrome, syndrome X, insulin resistance syndrome, obesity syndrome, and Reaven’s syndrome. • Metabolic syndrome is a set of risk factors that includes: – abdominal obesity – a decreased ability to process glucose (insulin resistance, hyperinsulinism, glucose intolerance) – dyslipidemia (unhealthy lipid levels - á VLDL, â HDL-cholesterol, á small, dense LDL) – and hypertension. • Patients with this syndrome are at increased risk of developing cardiovascular disease and/or type 2 diabetes. Key products of Adipocytes: Pathophysiology Free Fatty Acids (FFA), Cortisol, Leptin, • Insulin resistance Angiotensin, estrone (a type of oestrogen), – Major contributor is compliment factors, resistin, PAI-1 – Plasminogen overabundance of circulating activator inhibitor type I, TNF-α – Tumour necrosis fatty acids from adipose tissue factor-α, IL-6 – Interleukin-6, MCP-1 – Monocyte – FFA also derived from lipolysis of chemoattractant protein-1 triglyceride-rich lipoproteins in tissues by action of lipoprotein Consequences of Free Fatty Acids released from lipase expanded adipose tissue mass – Insulin inhibits lipolysis in adipose tissue • Liver - áin glucose, triglycerides & VLDL • Obesity • FFA - â insulin sensitivity in muscle • Dyslipidaemia • á Circulating glucose – hyperinsulinaemia – – in FFA to liver, es prodn of apo Bresults in á Na reabsorption and á sympathetic containing triglyceride-rich VLDL NS activity leading to hypertension • Glucose intolerance – Defects in insulin action lead to inability of insulin to suppress glucose production by liver, and mediate glucose uptake in muscle & adipose tissue • Hypertension – Leptin thought to play key role in elevation of sympathetic activity in obesity

– –

• • •

• •

Leptin áes sympathetic vasomotor tone indirectly via a baroreflex mechanism Renin-angiotensin system – adipocytescontain all components of the system which is upregulated in obesity Fasting reduces circulating leptin levels Inflammatory markers Insulin, glucocorticoids, oestrogens • Adipocytes & monocyte-derived macrophages - á induce leptin synthesis. secretion of IL6 &TNF-α Muscle • This á insulin resistance & lipolysis of – Leptin appears to increase triglycerides to FFA substrate oxidation and move • áCytokines in circulation may enhance hepatic fuels towards utilization and glucose prodn and VLDL by liver and insulin away from storage resitance by muscles – May help in reducing lipotoxicity • Cytokines & FFA also á prodn of fibrinogen and and help in increasing tissue plasminogen activator inhibitor 1 (PAI-1) by liver insulin sensitivity in obesity and Type II diabetes Liver – Leptin seems to effect gene expression of enzymes involved in gluconeogenesis Pancreas – Leptin appears to act at different intracellular levels to exert a physiological long-term control of insulin secretion from pancreatic beta cells.

Treatment – treat the underlying cause e.g. with diet and lifestyle changes. Then treat individual pathologies e.g. using a statin or metformin for weight loss or use aspirin to reduce clotting time.

Malnutrition and Nutrition Lectures

WATER Nutrition support should be considered for. • BMI <18.5 • Unintentional weight loss >10% (3-6 months) or BMI <20 and unintentional weight loss >5% Nutrition support for those at risk of malnutrition • Eaten little or nothing for >5days • Likely to eat little for next 5 days • Unable to take in nutrients properly • Those with increase nutritional needs White Paper 2006 • MOTs introduced at birth, 11 , 18, birth of first child, 50. • Assess lifestyle + family history • Diet, smoking habits, weight • If a high risk get a personal trainer (yeah whatever Tony and Gordon!) • To create shift away from emergency medicine and towards prevention. Basic Nutritional Requirements (Water) • Water 60-70% body weight. It is intracellular (2/3 body water or 40% body weight) or extracellular (1/3 body water or 20% body weight) • Provides constant external environment for cells • Plasma water (5% body weight) • Interstitial (interstitial/lymph 10%, bone and connective tissue 4%, transcellular 1%) • Third space – (oedema, subcutaneous) Basic Nutritional Requirements (Electrolytes) • Intracellular water K+ major cation = 150mEq/l, Na+ = 12mEq/l • Anions include Cl- and HCO3- plus phosphates, organic anions and proteins. • Extracellular water Na+ major cation = 145mEq/l, K+ 4mEq/l) • Cl- and HCO3- major anions • Plasma has more proteins than interstitial. Gradients between extra and intracellular maintained by Na+/K+ ATPase Plasma water volume is needed to • Maintain organ perfusion • Transporting O2 + Nutrients to cells. • Excreting urea and salts • Regulating body temp • Communication between organs (hormones, cytokines, neurotransmitters) Fluid balance maintained by • Thirst (input) Stimulated by high plasma osmolarity or hypovoleamia (renin) • Kidneys (output) Decreased plasma volume or increased plasma osmolality stimulates ADH leading to re absorption of water. • Normal Osmotic pressure of body fluids = 285 mosmol/l Dehydration • Severe diarrhoea, burns • Prolonged exercise • High temp • Diuretics • Diabetes, Diabetes insipidus, Nephrogenic Diabetes insipidus, Hypercalaemia, HONK, DKA. • Failure to drink. Oedema • Congestive heart failure – Cannot pump blood as fast as it comes in from the veins so fluid leaks into tissues. • Starvation – Liver cannot synthesise albumin, plasma pressure drops fluid passes from vascular system to extracellular spaces (kwashikor)



Nephrotic Syndrome! Membranous Glomerulinephritis Food 70 kg man Body composition • Water (63%) • Fat 12 kg (17%) • Protein 12kg (17%) • Calcium 1kg • Adipose tissue 28% • Skin 6% • Muscle 37% • Bone 14% Energy from food essential for • Basal metabolic rate (BMR) energy needed for vital functions, electrolyte equilibrium, cell and protein turnover, respiratory function, cardiovascular function = 60-75% daily energy intake • Physical activity usually 10-15% energy, but can rise to 70% of energy in heavy manual work or competition athletics • Thermogenesis (Metabolic Heat Generation) Isometric (muscle tone in sitting+standing) Dynamic (heat production without work), Psychological (anxiety or stress), Cold induced (shivering or non shivering, to keep warm) Diet induced (following eating) Drug induced (caffeine, nicotine, alcohol all stimulate thermogenesis) 1.Energy Balance • Change in energy stores = energy in – energy used • Energy balance (weight) generally occurs over 1-2 weeks (not day to day) • Most peoples weight varies a few Kgs over a decade. • Food intake varies around social and cultural events, psychological state, illness. 2.Energy Balance • Blood glucose homeostasis is maintained in the short term (hour to hour) • Hepatic glycogen stores maintained medium term (day to day) • Fat stores and protein compartments maintained in the long term (weeks, months, years) Energy in food • Carbohydrate (small body store of glycogen in liver and muscle, accurate, autoregulation) • Protein (moderate limited store of protein in lean tissue, accurate autoregulation) • Fat (moderate to large unlimited store of triglcerides in adipose tissue, poor autoregulation) • Alcohol (no store) 5-10% lost in transformation Energy Measure: Joules or calories • 1 Joule (J) = the energy used when (1kg) is move 1 metre (m) by the force of 1 Newton (N). • 1 Calorie (cal) = the energy needed to raise 1 gram of water from 14.5 to 15.5 d celcius. • 1 Calorie = 4.184 joules Average daily intake in the UK: • Males= 9720 kJ (2313 kcal) Females= 6870kJ (1632 Kcal) • 1 gram of carbohydrate produces 16kJ (3.75 kcal), or protein produces 17kJ (4kcal) fate provides 37kJ (9Kcal), alcohol provides 29kJ (7kcal) • 142 kcal in banana, 704 kcal in cheeseburger

Carbohydrate • Starches provide main source of food in diet plus metabolic fuel to all tissues after a meal. • Starches are precursors for ribose, de oxyribose, glycoproteins, glycolipids. • Via pyruvate are precursors for non essential amino acids and fatty acids. • Metabolised via gylcolysis to pyruvate • Stored as glycogen in the liver and the muscles.

Glucose and fasting (regulated by insulin and glucagon) • Blood glucose maintained at 3-5.5 mmol/l • Short term fast glycogen used to maintain glucose (12-18 hours) • Free fatty acids (adipose) and ketone bodies (liver) released. • Moderate fast gluconeogenesis maintains blood glucose (from amino acids and glycerol Dietary fibre • Carbs are not digested in the small intesting • Main energy source for colonics microbiota • Increased faecal bulk via binding of water and microbial mass

• •

Fermentation products (proprionate and butyrate) are energy soruces for colonic epithelial cells and influence peripheral metabolism) Soluble = fruit, veg, oats, Non-soluble = bran from cereal

Fat • • • • • • • • •

Stored for later energy (triacyl glycerides) Structure cell membranes Cell signalling role Help absorption of fat soluble vits Pre cursors of hormones and other mediators Saturated and trans fats lead to increased (LDL) cholesterol Mono- saturated and polyunsaturated fats have neutral or positive effects on the CVD risk factors and tend to reduce LDL slightly. Reducing saturated and trans fats is more useful than reducing total fat intake/ Could be special benefits from n-3 (omega 3) fatty acids.

Protein • Polymers of the 21 amio acids (9 essential) folding produces structures that act as receptors, transport of other structural proteins and enzymes. • Adults are in overall nitrogen equilibrium, intake matches excretion, in growth or recovery from loss we are in positive equilibrium. • Negative equilibrium = trauma or inadequate diet

Alcohol = no nutritional requirement!

• • • • •

Overall energy contribution is 6.5% in men and 3.9% in women. Can be up to 60% in alcoholics leading to nutritional problems in 50% alcoholics. Alcohol limits ! 21 units per week in men and 14 units in women. 1 unit = single pub measure of spirit, small glass of sherry 50ml, small glass of wine 125 ml, ¼ pint of strong lager, ½ pint of ordinary strength lager beer or cider, 2 pints (1136ml) of low alcohol lager beer or cider. Methanol oxidation produces acetaldehyde which is toxic. Associated with 60-120 pathologies due to Adduct formation, Changes in protein, carbohydrate and lipid Membrane dysfunction Impaired immune status Altered gene expression Oxidation Altered intracellular signalling Vitamins Water Soluble Vitamins = B1 – thiamine, B2 ribaflavin riboflavin, Niacin, B6, Folates, B12, C- (ascorbate) , Pantothemic acid, H-Biotin Fat soluble vitamins A- retinoids & carotenoids, D cholecalciferol & ergocalciferol, E tocopherols, K- phylloquinone & menaquinones

Water soluble vitamins • B1 (thiamine) precursor of thiamine diphosphate (coenz in metabolism of CHO, fat, alcohol) & triphosphate. (activates chloride channel in nerve membrance). • RNI= 0.4mg/1000kcal children 0.5-0.7 mg/d, men=1 mg/d, women 0.8 mg/d • <10% population had intakes 2oomg d causes vasodilation.



Food source meant and maize products, cereal products, vegetables, milk and dairy products, drinks. Meat and fish. B6 (pyridoxine) • Involved in amino acid metabolism, co factor for glycogen phosphorlase, haem synthesis, modulates steroid hormone action, regulates gene expression. • RNI relate to protein intake 15ug/g protein, Children 0.7-1.0 mg/d niacin equivalents, men 1.4mg/d, women 1.2mg/d. • 20% women, 6% men in UK50mg/d can cause sensory neuropathy, numbness, weakness. • Food sources cereals, meat, drinks, potatoes, milk and dairy products. B12 (cobalamin) • Group of cobalt enzymes, 3 cobalamin dependent enzymes (isomerisation, folate enzymes and synthesis of methionine from homocysteine), requirements RNI: children 0.5-1 ug/d, Men 1.5 ug/d, women 1.5ug/d, extra 0.5ug/d lactation • 4% of teenagers <1% adults and intakes


Toxicity High intakes taken to prevent colds (Cochrane review – no effect), Cessation after long duration may cause rebound scurvy, Increased risk of oxalate stone formation, diarrhoea

Biotin (H) • Cofactor for enzyme systems including gluconeogenesis and fatty acid synthesis • Requirements – no RNI defined • Intakes of 10-200μg/d thought to be safe and adequate • Stored in the liver, deficiency unlikely • Deficiency in experimental animals is teratogenic • Raw egg (avidin) binds biotin and prevents absorption, but does not occur with cooked egg • •

Food sources – main UK contributors, Cereal products (23%), Drinks (21%) – beer and coffee, Milk and dairy products (16%), Eggs (15%), Meat and meat products (11%) Good sources include (made by yeasts and bacteria, widely distributed in plant and animal tissue):Liver and kidney, yeast, Nuts & pulses, Wholegrain cereals, Eggs

Fat soluble Vitamins Vitamine A Retinoids and carotenoids • Retinol & carotenoids (pro-vitamin A) • 6μg β-carotene has vit A activity of 1μg retinol • Needed for normal tissue development and differentiation, also for night vision (rhodopsin in retina) • Requirements, RNI: Children 1-10yrs 400-500μg/d, 11-14yrs 600μg/d , Men 700μg/d, Women 600μg/d, Extra 100μg/d pregnancy, 350μg/d lactation • 2.5% of adults
Vitamin D – • Cholecaciferol (D3), ergocaciferol (D2) • Most produced by UV on 7-dehydrocholesterol in skin • effects on calcium homeostasis and bone synthesis • Requirements, RNI: Children <6mo 8.5μg/d, 7-48mo 7μg/d , 4-64 yrs nil (so long as some sun exposure) , 10μg/d pregnancy, or >64yrs, or no sun exposure Average dietary intakes in adults 3.1 to 3.8μg/d



Deficiency – rickets (skeletal deformity, pain, weakness) in children, osteomalacia (bone pain, muscle weakness) in adults, osteoporosis, Little or no skin exposure to sunlight, Conversion reduced (dark skin, older people), Requirements high (children, pregnancy) Vitamin E • Tocopherols and tocotrienols, α tocopherol is most active • Important fat soluble antioxidant – works in lipid membranes • Requirements, no RNI:Men >4mg/d , Women >3mg/d , But may need 0.4mg/g of PUFA, suggesting 6mg/d for women and 8mg/d for men • Average dietary intakes in adults 8.6 to 11.7mg/d • Deficiency – rare, only identified in premature babies, could be seen in those with high PUFA intakes or fat malabsorption • Food sources – main UK contributors Margarines and fat spreads (20%)Fats in potato or cereal dishes, Meat, fish and eggs (20%) • Good sources include:Vegetable oils, Fortified margarines & spreads • Toxicity Few ill effects seen from high intakes (up to 3.2g/d), But safety of high intakes to be established Vitamins K • Naphthoquinone: K1 phylloquinone; K2 menaquinone • Important for formation of prothrombin and factors necessary for blood coagulation • Requirements, no RNI: 1μg/kg body weight • Dietary intakes in UK unknown, but may be around 100 μg/d or 300-500 μg/d (varied est.) in USA • Deficiency – rare, may occur in newborns as stores are low and gut sterile, Vit K given to new babies in the UK, May also occur in those with altered lipid absorption, Leads to prolonged clotting times • Food sources – main UK contributors unclear • Good sources include: Green leafy vegetables, Soybean oil, Beef liver, Dried seaweed Menaquinones are synthesised by gut bacteria, but not clear how well they are absorbed • Toxicity Few adverse effects reported

Minerals and trace elements • Only required in small quantities, role and requirements not clear, difficult to assess amount consumed as foods very with soils, animal feeds, species, ripeness, food processing. • Difficult to estimate absorption due to dietary source, haem versus non haem, minerals may compete for same absorption site (iron and zinc), absorption decreases with age, Level of intake increases at low intakes, physiological need (depletion can increase absorption. • Assessment of mineral or trace element difficult as few simple reliable measures, serum levels limited where circulatin levels tightly controlled (eg calcium) or body stores severely depleted before circulating levels drop. • Circulating levels can be altered by disease or metabolic response to illness, infection or trauma. Calcium • Calcium needed for skeleton, 99% in bones & teeth as calcium salts (hydroxyapatite) • 1% in tissues or body fluids – membrane transport, muscle contraction, nerve transmission, blood clotting

• • • • •

Requirements highest when bones growing, RNI:1-3 yrs 350mg/d, 4-6 yrs 450mg/d, 7-10 yrs 550mg/d, 1118 yrs 1000mg/d boys, 800mg/d girls Adults 700mg/d (plus 550mg/d lactation), Additional needs in pregnancy met by increased absorption Need in older people to maintain vit D intake to offset reduced efficiency of calcium absorption 25% men & 48% women
Phosphorous • Present in all cells and linked to calcium and protein metabolism • 85% in bone (hydroxyapatite), • 15% phospholipids, nucleic acids, • oxygen and energy release to cells • Phosphate excretion (kidneys) maintains acid base balance • RNIs are the same as those for calcium expressed in mmols (exist in the body in equimolar amounts) • Adults 550mg/d (extra lactation 440mg/d) • Deficiency – myopathy, respiratory and cardiac failure, neuropathy, tissue hypoxia • unlikely except with altered pH, excessive phosphate loss (diabetic ketoacidosis or malabsorption), diuretic use, magnesium and aluminium antacids, poor parenteral feeds • Sources – most plant and animal foods Milk and milk products (25%), Cereal products (25%), Meat and meat products (20%), Veg and potatoes (10%) • Around 60% of dietary phosphorus is absorbed • Toxicity - >70mg/kg body weight can produce hyperphosphataemia Magnesium • Present in many enzyme systems inc decarboxylation, phosphate transfer and energy release • Vital roles in skeletal devpt, protein synthesis, muscle contraction, neurotransmission • RNI: • Men 300mg/d, women 270mg/d (extra lactation 50mg/d) • Deficiency – skeleton acts as Mg store, unlikely except with • high intestinal losses, increased renal excretion, diuretics, bowel cleansing solutions, plasmapheresis • Symptoms of hypomagnesaemia – muscle weakness, cramps, hypertension, cardiac arrhythmias • Sources – most plant and animal foods, also hard water, Bread and cereal products (30%), Drinks (20%) – beer and coffee, Veg and potatoes (16%), Milk and milk products (12%), Meat and meat products (10%), Around 20-30% of dietary phosphorus is absorbed (small intestine), Toxicity – excess excreted by kidneys, excessive intakes not absorbed • Risk in renal failure or adrenal insufficiency or enteral/parenteral overadministration • Around 20-30% of dietary phosphorus is absorbed (small intestine) • Toxicity – excess excreted by kidneys, excessive intakes not absorbed. Risk in renal failure or adrenal insufficiency or enteral/parenteral overadministration Sodium • Principal cation in extracellular fluid • Regulation of fluid balance, blood pressure, trans-membrane gradients • Requirement 69-460mg/d, intakes 2-10g/d • To reduce sodium from 3.6g/d to 2.4g/d (salt 9g/d to 6g/d) • LRNI: 575mg/d • Deficiency – extreme heat/exertion, Elderly – low intake plus reduced tubular reabsorption can lead to anorexia and confusion • To reduce sodium intake we need to: Reduce processed foods, Don’t add salt at table or in cooking • Sources – most plant and animal foods, also hard water, Manufactured and processed foods (60-70%), Addition of salt (table, cooking) (15-20%), Sodium naturally in foods (15-20%) • Concentrated sources are salty (ham, bacon, cheese, soups, smoked fish, products in brine etc) • Toxicity – high intakes emetic but can be fatal • Prolonged moderately high intakes appear to , Increase risk of blood pressure with age, Increase osteoporosis

Potassium • Predominant intracellular cation • Regulates acid base balance, fluid balance, muscle contraction, nerve conduction • 95% found intracellularly • RNI: 3500mg/d • 67% men, 93% women below RNI • May contribute to development of hypertension • Deficiency – muscular weakness, weakness of heart muscle, confusion • Toxicity – only with renal impairment – cardiac arrest • Good sources: fruit (bananas), vegetables, chocolate, coffee Zinc • Component of >70 enzymes (protein turnover) • 2g of zinc in body, 60% muscle, 30% bone • RNI: men 9.5mg/d, women 8.4mg/d (6.0mg/d extra lactation) • 30% men and women below RNI • Deficiency – evident in tissues with rapid turnover – mouth, skin, intestinal mucosa (loss of taste, reduced immune function) • Toxicity – >2g/d nausea, vomiting, interferes with iron, copper and manganese absorption • Good sources: widely distributed, better absorbed from animal sources (phytates in cereals) Copper • Component of oxidative enzymes • RNI: men and women 1.2mg/d, rise with age in children • Average UK intake 1.63mg/d men, 1.23mg/d women • Deficiency – rare except in genetic disorders • Leads to anaemia, neutropenia, bone changes • Toxicity – potential pro-oxidant effect • Good sources: shellfish, liver, nuts, cocoa (widely distributed) • 35-75% absorbed, reduced by phytate, zinc, iron, calcium, phosphorus

Selenium • Antioxidant effects – constituent of glutathione peroxidase and many other enzymes • RNI: men 75μg/d, women 60μg/d, extra 15μg/d lactation • Typical UK intake 62μg/d • Deficiency – Keshan disease (cardiomyopathy) • Toxicity – >750μg/d can cause problems • Good sources: widely distributed, affected by soil, low in UK – meats, fats, veg, cereals (Canadian wheat), fish • 55-65% absorbed Iodine • Works as part of thyroid hormones (thyroxine) • RNI: 140μg/d • Typical UK intake 243μg/d men, 176μg/d women • Deficiency – thyroid gland hyperplasia, goitre • In pregnancy – stillbirth, abortions, perinatal deaths, Infants and children – impaired brain development, cretinism (retardation, dwarfism, hypothyroidism) • Toxicity – hyperthyroidism, linked to thyroid cancer • Good sources: milk (iodine has risen with increased levels in animal feed), seafoods, seaweeds well absorbed, inhibited by various plant compounds (thiocyanates),

Carbohydrates Carbohydrates have the generic formula (CH2O)n or associated – n may vary e.g. ribose n=5, glucose n=6 – Single sugars (monosaccharides) may condense (loss of H2O) to give di-, oligo- or polysaccharides – They may contain other ‘active’ group substitutes such as –NH2 – They are central to intermediary metabolism and are the main energy source for most life forms – They may form parts of cell structures and may be covalently bound to proteins or lipids and they are part of nucleic acid structure Carbohydrate Isomers • There are two main optica isomer families D and L – This is based on the effect of sugars on rotation of plane polarised light – Natural sugars are largely D form • Monosaccharides can exist in ‘straight chain’ or ‘ring’ forms • Depending on position of an O=C- they may be aldehyde (reducing) or keto form -C(HOH)-CHO or –CO-C(HOH) • Relative location of –OH groups on adjacent carbons leads to differing chemical and biological characteristics Intermediary metabolism • Metabolic processes obey the laws of chemistry • They are: • Integrated • Compartmentalised • Within the cell • Within organs • Within the whole body • Controlled • Hierarchical Carbohydrates are central to intermediary metabolism In particular they are the means through which energy is made available Without such available energy the improbable organisation that is the basis of life cannot be sustained Carbohydrate Metabolism • Energy release – The glycolytic (Embden-Meyerhoff) pathway – The tricarboxylic acid (Kreb’s) cycle – Energy storage – The Pentose Phosphate pathway – Glycogen – Interconversion to Triglyceride Glycolytic Pathway • Glucose =>> Pyruvate • Sequential internal rearrangements of a 6 carbon skeleton followed by splitting into two three carbon units and further internal rearrangement • Located in the cytoplasm Glycolysis • Essentially oxidation/reduction C6H12O6 =>>>> 2 CH3-CO-COOH Glucose Pyruvate

Note there are four hydrogen atoms ‘missing’ • Pyruvate is central to several metabolic pathways Glycolysis is energy releasing • The energy is in the form of a high energy phosphate bond in ATP • ATP can be used to drive biochemical processes in the ‘wrong’ direction energetically – e.g. protein synthesis Glycolysis also produces NADH • NADH is part of a redox couple NAD+  NADH • NAD+ is regenerated from NADH 1. Though the electron transport chain 2. By reduction of pyruvate to lactate CH3-CO-COOH + NADH  CH3-CH(OH)-COOH + NAD+ Glycolysis • The full rearrangement Glucose  2 Lactate is anaerobic (i.e. does not require oxygen) • This pathway produces 2 ATP/mole glucose Glycolysis MAIN STEPS 1.Phosphorylation • Phosphorylation is the first step • It uses ATP Glucose + ATP → Glucose-6-phosphate + ADP This ‘primes’ the pathway 2. Isomerisation Glucose-6-phosphate → Fructose-6-phosphate This changes the aldehyde (-CHO) to an alcohol (-CH2OH) at carbon 1 This is approximately energy neutral. Reactions like this could be driven by increasing the substrate (G-6-P) or reducing (removing) the product (F-6-P) However, rates of biological reactions are more frequently moderated by enzyme activity 3. Second phosphorylation by ATP F-6-P + ATP → Fructose 1,6 diphosphate + ADP Note that the previous isomerisation step made this phosphorylation possible by producing the –OH on carbon 1 of the sugar 3. The enzyme phosphofructokinase that catalyses this step is a control point Low activity will result in G-6-P being available for: a) The pentose cycle b) Glycogen synthesis

4 Hexose to triose The six carbon skeleton is split into two three carbon fragments by aldolase Dihydroxyacetone P F 1,6 DiP Glyceraldehyde-3-P Note the isomerisation of the triose phosphates. This is energetically an unfavourable reaction which is driven by the formation of F 1,6 DiP and the oxidation of glyceraldehyde-3-P 6. ‘Recovery’ of ATP The phosphate anhydride bond has sufficient energy to transfer to ADP to give ATP 1,3 DPG +ADP → 3 phosphoglycerate +ATP Since two molecules of 1,3 DPG have been formed 2 molecules of ATP are recovered. 7. Generation of further high energy phosphate bond

Two steps: Isomerisation – moves phosphate from carbon 3 to carbon 2 3 phosphoglycerate → 2 phosphoglycerate Dehydration – produces phosphoenolpyruvate CH2=C(0~P)-COO7. Generation of further high energy phosphate bond PEP +ADP → Pyruvate +ATP Since there are two molecules per molecule of glucose, there is a net gain of 2 ATP For each glucose there has been the production of: – 2 ATP – 2 Pyruvate – 2 NADH Trycarboxylic Acid • This is the energy ‘power house’ • Further oxidises carbon skeleton to produce CO2 • Based in mitochondrion • Linked to ATP formation through: – The electron transport chain – The proton pump – Chemiosmotic coupling Trycaboxylic Acid Cycle 1.Pyruvate ‘enters’ the cycle 2As Acetyl CoA formed by oxidation (More NADH produced) 3.Condenses with oxaloacetate (2 carbon) (→ citrate (3 carbon) hence ‘citric acid cycle’) 4.Three oxidations produce NADH and one produces FADH2 which has a lower energy 5.The cycle regenerates oxaloacetate • Both oxaloacetate and α-Ketoglutarate are intermediaries in the pathway – These are two of several intermediaries that allow • Junctions with other pathways • Replenishment of the TCAC intermediaries • The membrane of the mitochondrion separates the TCAC from the cytoplasm which aids control of overall metabolic processes Pentose Phosphate Pathway • Glycolysis and the TCAC break down carbohydrate and produce energy • PPP produces NADPH • NADPH – Can pass ‘H2’ to NAD+ (→ TCAC) – Mostly used in synthesis e.g. formation of fatty acids (i.e. energy storage) • For each G-6-P 12 molecules of NADPH are produced • The pathway is cytoplasmic • NADPH can be used to reduce carbon (especially carbohydrate derived) skeletons to give fatty acids • Hydrogen in NADPH can also be transferred into the mitochondrion by a shuttle across the mitochondrial membrane. This can then be used by the electron transport chain to produce ATP Storage • Glygogen • Liver and muscle • Polysaccharide • Branched chain • Many hydroxyl groups therefore hydrophilic • Potentially ‘disruptive’ Pyruvate Metabolic Junction • Pyruvate is at a branch point – Acetyl-CoA → TCAC or Fatty acid synthesis – Lactate (anaerobic metabolism) → NAD+ regeneration – Amino acid synthesis (pyruvate → alanine) – TCAC intermediary synthesis • Malate

• Oxaloacetate The fate of intermediaries such as this depends on the prevailing conditions – Low ATP and high oxygen → TCAC (aerobic catabolism) – Low ATP and low oxygen → High NADH → lactate production – Low TCAC intermediaries → oxaloacetate – Amino acid excess or anabolic state → amino transfers Carbohydrate metabolism and the Disease State • Interactions of pathways are all essentially explicable as ‘simple’ chemical interactions • The variety of the interactions leads to complexity • Interactions can be considered at varying levels from cellular, through tissue to whole body • From a metabolic viewpoint disease is a disturbance of a dynamic equilibrium • What we observe is the sum of the disturbance and the response to the disturbance • When you are trying to understand findings in the disease state remember that you may be observing a consequence of a disturbance rather than the disturbance itself • Interactions of pathways are all essentially explicable as ‘simple’ chemical interactions • The variety of the interactions leads to complexity • Interactions can be considered at varying levels from cellular, through tissue to whole body An Example of Interatcion 2,3 DPG • When the body requires energy it also requires higher oxygen flow • Oxygen availability at tissue level depends on - Rate of delivery - Oxygen tension gradient - Avidity of binding to haemoglobin • Although this example uses only a side reaction of the glycolytic pathway, it demonstrates how physiological control in one tissue may have impact in other tissues and how an abnormal state (such as low phosphate secondary to antacid use) may lead to clinical signs or symptoms • Note also that phosphate depletion can occur in a number of other conditions including DM Given the integration of metabolism what would you expect to happen in DM •

Protein Synthesis General  We have 20 amino acids to synthesise proteins, peptides and neurotransmitters ( general formula CH.COOH.NH3-R)  10 are essential or semi essential – they cannot be synthesised  Some undergo modification after protein transcription o Proline > Hydroxyproline in collagen  When proteins are broken down ( 1-2% day) not all amino acids can be reused  The availability of essential amino acids may limit rate of protein synthesis  Animal protein provides amino acids in about correct proportions ‘first class protein’ Synthesis  Non-essential amino acids can be synthesised – carbon skeletons taken from other AAs or intermediates from other metabolic pathways, the amino group from other AAs by transamination.  Essential AA derived from diet  Cannot be stored

 Used for synthesis of proteins, other functional molecules, generating energy, conversion to fat / glucose. Protein turnover  Determined by locally functional demands on tissue – plasma cells (Immunoglobulins), osteoblasts (collagen), liver (acute phase proteins, clotting factors)  Balance between anabolic (insulin, GH, testosterone) and catabolic (cortisol, adrenaline, glucagon) stimuli  in the catabolic state – protein is lost from tissues (skeletal muscle, bone, gut)  protein synthesis may be impaired by ↓ immunity, loss of organ function Amino acid uses  Used as fuel when they are plentiful or if other sources are depleted  Form glucose (glucogenic)  Make fatty acids (ketones – ketogenic) Nitrogen  Proteins are the main source of N in the diet  The N is removed from the amino acid carbon skeleton before the AA is metabolised  Most of the N is incorporated into urea (80%) and some is lost as urate or ammonia  negative N balance = urine N excretion > N itake = net loss of body protein PKU  Inherited disorder (AR) 1/16000  Defective phenylalanine hydroxylase  High plasma phenylalanine is neurotoxic  Abnormal metabolites accumulate  Low phenylalanine diet with tyrosine supplements  Clinical features: irritability, poor feeding, vomiting, fitting in 1st few weeks of life, mental retardation, eczema, ↓ melanin formation in skin (fair hair, blue eyes)  Test detection of ↑ phenylalanine in blood spot  Management reduce plasma phenylketonuria by dietary control Urea cycle  Before utilisation of the carbon skeletons the N is lost by transamination  Pyruvate accepts N → alanine which transfers the N to the liver → transamination of a-ketoglutarate and oxaloacetate, N enters the urea cycle.  Urea synthesis only takes place in the liver – ammonia may be ↑ in liver disease and ↑ in neonates in severe illness – ammonia is very toxic! (Each step has a specific enzyme, and these may be defective) Defects  Enzyme defects in urea cycle are rare  When N delivery to cycle exceeds capacity plasma ammonia rises → results in a child with variable symptoms  Disproportionate severity of minor illness  Cycle intermediates pre-block found in urine

 Treatment minimises protein load and provides benzoic acid to enhance ammonia excretion: Benzoate + glycine -> hippuric acid

Introduction Triacylglycerol stores in adipose tissue serve as the bodies major fuel reserve. Fatty acids are easily mobilised to provide energy during prolonged starvation or exercise. > Oxidation yields of fat are 9kcal/g compared to only 4kcal/g for proteins and carbohydrates. > Fatty acid breakdown is the process by which a molecule of fatty acid is degraded by the sequential removal of two carbon units, producing acetyl CoA which can then be oxidised to CO2 and H2O > This process occurs in many tissues especially the liver and muscle, although certain tissues are unable to oxidise fatty acids such as the brain, adrenal medulla and RBC’s because they lack the necessary enzymes. > There are four stages to lipid breakdown: Lipolysis, activation of fatty acids, transport into mitochondria and β oxidation. 1) Lipolysis >The initial event in the breakdown of fat is the hydrolysis of Triacylglycerol stores in adipose tissue to glycerol and three fatty acids. Triacylglycerol→ Glycerol + 3 fatty acids > The glycerol produced cannot be metabolised by the adipose tissue as it lacks the enzyme glycerol kinase so it is transported to the liver where it is phosphorylated, either to be used again to make Triacylglycerol or to be converted into DHAP (dihydroxyacetone phosphate), a glycolytic intermediate. > The three fatty acids produced are re-esterified to Triacylglycerol in the adipose tissue or travel in the blood to be taken up by the cells for oxidation

2) Activation of fatty acids to fatty acyl CoA > Before they can be oxidised, fatty acids are activated by attachment to CoA enzyme to form acyl CoA molecules. > Fatty CoA synthetase (thiokinase) activates fatty acids by attaching them to CoA (coenzyme A: nucleotide containing pantothenic acid, which is an important coenzyme in the Krebs cycle and in the metabolism of fatty acids)

> The reaction occurs at the outer mitochondrial membrane and requires ATP which is hydrolysed to AMP and pyrophosphate (PPi), breaking a high-energy phosphate bond. > The reaction is made irreversible by the rapid hydrolysis of pyrophosphate to two free inorganic phosphates by pyrophosphatase, consuming a second high-energy phosphate bond > Therefore the activation of a fatty acid consumes 2ATP equivalents. > Fatty acids are non-polar molecules so can easily diffuse out of cells, but the attachments to a polar molecule such as CoA ‘traps’ the fatty acid inside. 3) Transport of fatty acyl COA into mitochondria

> The activation of fatty acids occurs in the cytosol but the enzymes for β oxidation are in the mitochondrial matrix. > The inner mitochondrial membrane is relatively impermeable to long-chain acyl CoA molecules, so a special transporter system is required to carry fatty acids across. > The carnitine shuttle consists of three enzymes: a translocase and two carnitine acyl transferases (CAT I&II) 1) The acyl group is transferred from CoA to carnitine by carnitine acyl transferase I, an enzyme found on the cytosolic side of the inner mitochondrial membran 2) Acylcarnitine is transported across the membrane by the translocase to the mitochondrial matrix. 3) The acyl group is transferred back to CoA by carnitine acyl transferase II, located on the inner surface of the inner mitochondrial membrane 4) Carnitine is returned to the cytosolic side in exchange for another molecule of Acylcarnitine Pyrophosphatase Water

Activation ATP Fatty acid +CoA

AMP +PPi

Fatty acyl CoA synthetase

2Pi

Acyl CoA

1

Acyl CoA Carnitine Shuffle

Carnitine

4 Carnitine Acyl CoA

CoA

CAT I

Acylcarnitine

2

Translocase CAT II

3

Acylcarnitine CoA

β oxidation Inner Mitochondrial membrane

Muscle or liver cell

4) β oxidation > Fatty acids are degraded by a cyclical sequence of four reactions: oxidation ,hydration, oxidation and thiolysis. > This results in shortening of the fatty acid chain by two carbon atoms per sequence. > The two carbon atoms are removed as acetyl CoA. > Each round of β oxidation produces one molecule each of FADH2, NADH and acetyl CoA.

> To give an example we will use the fatty acid palmitate which is the most common fatty acid found in animals and plants. The β oxidation of palmitate requires 7 cycles producing 7FADH2, 7NADH and 8 acetyl CoA. ATP Yield > The activation of palmitate to palmitoyl CoA consumes 2 molecules of ATP. β oxidation generates: - 7FADH2, which are oxidised by the electron transport chain to give 10.5 ATP. - 7 NADH, which are oxidised by the chain to give 17.5 ATP - 8 acetyl CoA, which are oxidised by the Krebs cycle to generate 80 ATP (oxidation of each acetyl CoA by Krebs cycle yields 10ATP) > Therefore the total energy generated from the oxidation of a molecule of palmitate is 106 ATP. Regulation of Lipid Breakdown > The control of lipid breakdown is exerted at three levels: Lipolysis, carnitine shuttle and β oxidation. (See earlier) Control of Lipolysis > Hormone sensitive lipase is regulated by reversible phosphorylation. > Adrenaline during exercise, and glucagon and adrenocorticotropic hormone (ACTH) during starvation, activate adenylate cyclise, which increases the levels of cAMP. > This activates a cAMP-dependent protein kinase, which phosphorylates lipase, activating it!! > The same cAMP-dependent protein kinase also phosphorylates acetyl CoA carboxylase, inhibiting it, therefore stimulating Lipolysis but inhibits fatty acid synthesis. Carnitine shuttle

> Malonyl CoA inhibits carnitine acyl transferase I (CAT I), thus inhibiting the entry of acyl groups into mitochondria. > An increase in malonyl COA is produced during fatty acid synthesis and ensures that newly synthesized fatty acids are not transported into mitochondria for oxidation as soon as they are made. Inhibition of β oxidation by NADH and FADH2 > The oxidation requires a supply of FAD and NAD+, which are regenerated via the electron transport chain. > The enzymes of β oxidation have to compete with the dehydrogenase enzymes of the Krebs cycle for NAD+ and FAD because both pathways are usually active at the same time. Emergency Treatment of HONK and Ketoacidosis

Causes = undiagnosed diabetes, stopping insulin therapy, infection UTI or Upper Resp, Drugs cocaine! Pathogenesis • Results from uncontrolled catabolism due associated with insulin deficiency which causes increased hepatic glucose consumption, reduced peripheral uptake by muscle and increased lypolysis. • Second important feature is fluid depletion resulting from osmotic dieresis. Blood glucose maybe 1020mmol/L in some patients, particularly in children but usually >20mmol/L • Lypolysis leads to excessive increased fatty acids. Excess of counter regulatory hormone leads to excacerbation of insulin deficiency and the two together lead to increase in ketone bodies from hepatic mitochondria. • Ketones are in urine and cause a smell in breath like nail varnish remover or acetone • This leads to a metabolic acidosis. • Respiratory compensation leads to hyperventilation “air hunger” • Vomiting exacerbates dehydration and loss of electrolytes from polyuria • Dehydration inhibits renal excretion of hydrogen ions and ketones, increasing acidosis. Symptoms and signs • Classic are nauseas and vomiting with abdominal pain • Polyuria, polydipsia, weight loss • Dehyration, tachydardia, hypotension, warm dry skin, hyperventilation, confusion, coma. Investigations • Blood glucose, FBC, ABGs (metabolic acidosis), Urine dipstick (ketones, pyuria, bloodprotein) • Bacteriology: culture from blood, urine, and swab from any infection • ECG peak in T waves (hyperkalaemia) flat T waves hypokalaemia • Chest radiograph infection of cardiac failure • CT head if cerebral oedema a concern Management • Soluble insulin (6-10 U/h) is given as IV infusion hourly or IM injections.

• •

Replace fluids usually lost about 5 litres. Use normal saline. Replace electrolyte loss: Potassium levels must be monitored as person can have a vessel hyperkaleamia even though they may have total body loss of potassium. IV potassium is 40mmol/L when serum postassium in the normal range. • Restore acid base balance. Both fluid replacement and insulin will restore this. Bicarbonate is controversial and only if considered if pH is less than 7.0 and best be given as isotonic solution (1.26%) • Detect underlying cause. Physical exam and tests may find underlying infection • Once blood glucose falls to 10mmol/L IV dextrose and insulin (3U/h) are started until patient can eat. At this stage S/C insulin is started. IV insulin infusion is stopped and similar amount is given as 3 injections of soluble insuluin s/c at meal times and a dose of intermediate acting insulin at night. Complications • Coma, cerebral oedema, hypotension, hypothermia. Hyper osmolar non ketotic hyperosmolar states HONK Severe hyperglycaemia may develop without significant ketosis. These patients have type 2 diabetes, usually adults and often previously undiagnosed diabetes. • Precipitating factors include energy drinks, concurrent medication such as thiazide diuretics or steroids, and intercurrent illness. • The biochemical differences between HONK and ketoacidosis are caused by age: extreme dehydration characteristic of non ketotic coma maybe less severe thirst and more severe renal dysfunction.Insulin deficiency: Modest insulin deficiency found in type 2 diabetes means that endogenous insulin levels are sufficient to inhibit hepatic ketogenesis. • Clinical features Typically present with severe dehydration, stupor and coma. Impairment of consciousness is related to the degree of hyperosmolality. Look for an underlying illness. Patients are particularly prone to arterial thrombosis, leading to cerebrovascular accidents, myocardial infacrtion or arterial insufficiency. • Treatment: Osmolality adjustment. Plasma osmolality is usually extremely high, it must be estimated and monitored (normal range 275-300 mmol/kg). • Fluid replacement Normal saline. Dont use 0.45% as rapid dilution of blood leads to cerebral damage. • Careful insulin use : Many patients are very sensitive to insulin and glucose concentration may fall rapidly, resulting in cerebral oedema. If the glucose falls, a smaller dose of insulin maybe sufficient. • Anticoagulation prophylaxis. In view of the propensity of patients with HONK for arterial thrombosis, such therapy is particularly important. • Prognosis Mortality in HONK maybe as high as 20-30%. Most patients once recovered can be treated with tablets and diet or even diet alone. Nephrotoxic Drugs Patients with normal baseline renal function may develop renal impairment with wide variety of drugs. Typical bastards are: • Aminoglycosides = can cause tubular necrosis, especially in combination with diuretics!! Close supervision needed when prescribing aminoglycosides with any impaired renal function. • NSAIDS = acute or chronic renal failure. In vulnerable patients with comorbidities, the addition of NSAIDS is sometimes sufficient to precipitate ATN. Chronic usage can lead to glomerulinephritis or papillary necrosis. • Lithium = Acute lithium toxicity can be precipitated by many other drugs, including diuretics and NSAIDS. Severe toxic levels and deliberate overdoses can be treated by haemodialysis. Haemodyalysis clears the drug and reverses toxicity. Chronic usage = nephrogenic diabetes or chronic tubulointerstitial nephropathy. • Nephrotic syndrome = NSAIDs,Penicillamine, Gold. • Acute renal failure = ACE inhibitors, NSAIDS • Membranous Glomerulonephritis = Gold, NSAIDS, Captopril, Penicillamine. • Tubular Necrosis = Aminoglycosides, Contrast media and Ciclosporin. • Acute Interstitial Nephritis always drug related (REMEMBER RASH EOSINOPHILIA and ACHING JOINTS!!)  Causes Antibiotics = Cephalosporins, Penicillins, Sulphonamides, Vancomycin.

     • •

NSAIDS. Anticonvulsants Phenytoin and Carbamazepine. Diuretics Furosemide and thiazides Allopurinol Azathioprine

Distal tubular acidosis = amphotericin Hyporeninaemic hypoaldosteronism = ciclosporin

Glomerulonephritis third most common cause of end stage renal disease! Based on microscopy the disease can be focal (affecting some glomeruli) or segmental effecting affecting only part of the glomerulus or diffuse (affecting all glomeruli). •

Membranous Glomerulonephritis also known as epimembranous and extra membranous glomerulonephritis. • Epidemiology Approximately 25% adult glomerulonephritis in adults due to membranous glomerulopathy. Uncommon before age 30. Also most common cause of nephrotic syndrome of all glomerunephropathies. • Pathology: Approx 80% is idiopathic. Rest is caused by systemic disease, drugs, infection and malignancy. There is diffuse thickening of the walls of the capillary loop due to electron dense deposits. Characteristics spikes of basement membrane are seen on light and electron microscopy. • Clinical Features Patient presents with nephrotic syndrome. Less commonly asymptomatic proteinuria (frothy urine) or haematuria. On examination, hypertension maybe evident. • Investigations: U+Es Serum creatinine elevated. Urinanalysis maybe be positive for blood and protein. Renal biopsy and Screening for malignancy. • Management: Anticoagulation-Warfarin or heparin as high risk of thrombosis. Immunomodulation – prolonged doses of steroids or ciclosporin, chlorambucil or cyclophosphamide. • Prognosis: 25% patients develop spontaneous remission. At 10 years 25% of patients with idiopathic disease will progress to end stage renal failure and 25% will be receiving dialysis or will have died. Focal Segmental Glomerulosclerosis • Epidemiology FSGS is an increasing cause of nephrotic syndrome. • Pathology: The causes are primary and secondary (heroin use, reflux nephropathy, HIV, Obsesity) • Clinical Features They tend to present with nephrotic syndrome. May have haematuria or hypertension. • Investigations Renal biopsy • Management There is no real specific treatment. • Prognosis Progressive approx half of patients reaching end stage renal failure in 10 years. IgA nephropathy also known as Bergers • Epidemiology IgA nephropathy is most common cause of glomerulonephritis seen worldwide, found in up to 40% of renal biopsies. • Pathology: Abnormal amounts of IgA deposited in the renal mesagnium, but the aetiology is unknown. Granular IgA deposition leads to diffuse mesangial cell proliferation. IgA nephropathy is associated with immune related disorders such as RA, celiac disease and inflammatory bowel disease. • Clinical Features: Macroscopic or microscopic haematuria. Increasing urine screening in the general population is revealing more cases. • Investigations Urinanalysis dipstick testing positive for blood and protein. Proteinuria of nephrotic syndrome is rare, Serology ANCA, ant-GBM, autoantibodies, RA and complement levels. Renal biopsy indicated for patients with and elevated serum creatinine and or proteinuria .1g/24hrs. • Supportive management There are no proven ways of reducing production of the IgA. So good blood pressure control with ACE inhibitor is the main supportive treatment. Immunomodulation maybe used on rare occasions it is associated with a cresenteric glomerulonephritis and rapid decline in renal function. Plasma exchange, prednisolone (30-60mg/day) ciclosporin, azathioprine and cyclophosphamide. • Prognosis Majority of patients have a good prognosis. Progression is variable from mild benign to aggressive leading to dialysis within months.

Membranoproliferative glomerulonephritis (also known as MCGN type 1 and 2) • Epidemiology Increasingly common in the west. It can present in childhood or later as part of chronic immune disorder. • Pathology There are two types MPGN 1 which is characterised by splitting of the basement membrane giving “tram line” effect. Low C3 and normal C4 Primary or Secondary to (Cryoglobulins, SLE, Hep B and C, Sickle cell disease, CLL,malignancy). MPGN 2 seems to be a distinct entity with long segments of staining deposits (C3 dense deposit disease). Cause is idiopathic. Low plasma C3 circulating C3 nephritic factor and partial lipodystrophy. • Clinical features Patients may present with nephritic, nephrotic syngdomes or acute renal failure. • Investigations Urinalysis = protein and blood. 24 hour urine collection. Serology low compliment levels and C3 nephritic factor is present. Renal Biopsy confirms diagnosis. • Management Supportive treatment control of BP and Immunomodulation. • Prognosis Is poor 50% are in end stage renal failure requiring dialysis or dead by 5 years.

Minimal Change Disease (also known as lipoid nephrosis) • Epidemiology Most common cause of glomerulonephritis in children and accounts for up to one third of adult cases. • Pathology The cause of minimal change disease is unknown. Thought to be mediated by antibody mediated injury in genetically susceptible individuals in view of the association with atopy. HLA DR3 in Europe and DR8 in Japan. Only significant change on electron microscopy is fusion of the foot processes (podpcytes). • Clinical features Patients always present with nephrotic syndrome such as infection and thrombosis. Post-Streptococcal Glomerulonephritis (also known as diffuse exudative proliferative glomerulonephritis) • Epidemiology Rare in the West. More common in males and tends to occur in children. • Pathology Post streptococcal glomerulnephritis is an immune complex mediated glomerulonephritis that develops 10-14 days after a throat of skin infection with certain strains of b haemolytic streptococci. The histological appearance is diffuse proliferative glomerulonephritis and prominent polymorphic infiltration. • Clinical features Patiens tend to present with acute nephritic syndrome (PHAROAH! Proteniuria, Haematuria, Red cell casts, , Hypertension. This usually resolves but can rarely progress to cardiac failure, hypertensive encephalopathy or acute renal failure. • Investigations Anti-Streptolysin O titre. ASO titres are usually elevated. Plasma compliment levels Plasme C3 and C4 are low. • Management : Penicillin is administered for acute streptococcal infection. • Prognosis: Self limiting condition and the glom nephritis settles spontaneously within 1 or 2 weeks. Less than 5% of patients need temporary renal support but 20 % of adult patients are left with mild proteinuria or microscopic haematuria. Rapidly Progressive glomperulonephritis (also known as diffuse crescenteric glomerulonephritis) • Epidemiology: Uncommon • Pathology: Primary idiopathic or secondary to systemic disease (goodpastures disease, Wegeners, SLE,Henoch Schlonein purpura, IgA nephropathy, Post Strep Glom), Histology has presence of crescents and necrosis in the glomeruli. The cresents are cells, fibrin and collagen in the Bowmans space eventually this leads to irrerversible obliteration of the glomerular capillary tuft. • Clinical Features Haematuria, proteinuria with rapid deterioration of renal function to oliguric or anuric renal failure within days. • Investigations Urinanalysis, Serology, Renal biopsy (urgently!) • Management : Plasma exchange and Immunomodulation Methlyprednisolone (500 mg i.v. daily for 3 days) is administered to pulsed doses. • Prognosis In general poor many patients developing end stage renal failure. Henoch-Schonlein syndrome

• •

GFR •

Characterised by a skin rash, abdominal colic, joint pain and glomerulonephritis. The rash is purpuric type. Occurs in all ages and sexes but mainly disease of early childhood. Males twice as likely to get the disease. A recent history of infection often respiratory is common. The renal lesion is a focal segmental proliferative glomerulanephritis, sometimes with mesangial hypercellularity. Epithelial crescents maybe present

Each kidney has about 1 million nephrons and the measured GFR is the composite function of all nephrons in both kidneys and conceptually it can be understood as the clearance of a substance from a volume of plasma into the urine per unit of time. The ideal substance does not exist. The ideal characteristics being free filtration across the glomerulus, neither re-absorption from nor secretion into renal tubules, in a steady state concentration in the plasma, and easily and reliably measured.



Urea concentration is influenced more by dietary intake of protein, the state of hydration, liver function and various drugs.



Serum creatinine is a more reliable measure (and is universally used even though it fails to meet a few of the ideal criteria), but it is directly related to muscle mass i.e a small elderly lady may have normal serum creatinine with a markedly reduced (GFR). Changes in serum creatinine (especially a rise) can be a useful guide to deteriorating renal function; absolute values do not correlate with GFR.



It is also also important to realise that a significant rise in serum creatinine does not occur until the GFR is reduced to about 50% of normal. A number of formulas have been described to estimate GFR based on the serum creatinine and the characteristics of the patient (age, sex, weight, race). The formula that has been adopted in the UK, USA and many countries is the four variable Modified Diet in Renal Disease (MDRD) formula. It must be appreciated that this formula may not be as accurate in ethnic minority patients, in the elderly, in pregnant women, the malnourished, amputees, or in children under 16 years of age. Getting a value for GFR is useful though the value from the MDRD method is only an estimate whos accuracy diminishes as GFR exceeds 60mL/min and values should be viewed as having significant error margins rather than precise. Values can only be used when renal function is in a steady state i.e. not in acute renal failure. It is unwise to rely exclusively on the formula being between eGFR 60 and 89mL/min (CKD stage 2) because of its shortcomings, while values >90 mL/min should be reported not a precise figure. There is an urgency for better markers and better formulae.





• MDRD formula (conventional units) 170 * serum creatinine (mg/dl)-0.999*age-0.176*(blood urea nitrogen [serum urea])-0.17*albumin0.318*0.762 if female*1.18 if black Protein • Protein is normally present in urine in small quantities. Normal 80+/-25mg a day (<150mg is quoted as upper normal limit) Adolescents up to 300 mg/day Normal Protein Urine Content Consists Of • 60% from filtration - albumin- immunoglobulins • 40% originates from tubules So normal urine protein contains: 40% albumin 40% tissue proteins 5% immunoglobulins 5% light chains •

Abnormal Proteinuria Glomerular Permeability: Proteinuria due to increased glomerular permeability may lead to nephrotic range proteinuria (>3.5 g/day). - selective; mainly albumin (MW 69,000)

non selective; larger proteins (IgG 160,000 βlipoprotein 1, 100,000) Index of glomerular protein selectivity. Of clinical use in children with nephrotic syndrome. Increased permeability most common reason for proteinuria. Renal Tubular Re absorption Normally any filtered protein is reabsorbed,Tubular disease lead to reduced absorption (α2 and βglobulins),β2 microglobulins may be a marker of tubular diseases, Not nephrotic range proteinuria Alteration of plasma proteins • Immunoglobulin light chain in plasma cell disorders • Myoglobinuria/haemoglobinuria • Human albumin solution infusion Addition of proteins to tubular fluid • Immunoglobulin light chain in plasma cell disorders • Myoglobinuria/haemoglobinuria • Human albumin solution infusions

Table showing different ways to express urinary protein.

Urine Dipstick

Normal 0 Microalbuminuria 0

Albumin excretion rate (AER) (ug/min ; mg/24h) 6-20; 10-30 >20-200; 30-300

Trace Proteinuria Proteinuria Nephrotic

>200; >300 N/A N/A N/A N/A

Trace +,++ +++

Urinary albumin: creatinine ratio (mg/mmol)

Protein Urinary Protein (mg)/creatinine (mg/24hr) (mmol)

<2.5(m)<3.5(w) >2.5(m) >3.5(w) 15-29 N/A N/A

<15 <15

<150 <100

15-29 30-350 >350

150-299 300-3500 >3.5g

Ischemic foot – • Ischemia is when oxygenated blood supply does not meet demand • Damage is caused as a result of lack of oxygen+nutrients and a build up in waste products • Ischemia occurs most often as a result of arterial insufficiency • primary cause is atherosclerosis, it is caused by the accumulation of cholesterol, fatty deposits, cellular waste products, calcium and other substances lining the medium and large arteries • Other rarer causes include:giant cell arteritis, Buerger's disease Takayasu's disease • The risk factors for ischemia(atherosclerosis) include -increasing age, male sex ,smoking - two to three fold increase in risk, hypertension: 2.5 fold increased risk in men,3.9 fold increased risk in women diabetes two to three fold increase in risk, syndrome X hyperlipidaemia: a high cholesterol with HDL ratio gives highest risk, obesity • Investigations include – doplers, arteriography, claudication testing. • Treatment is removal of underlying cause and risk reduction- lifestyle changes, medication to include – aspirin, statin, anti hypertensives if applicable. Surgery so stenting or angioplasty if applicable • Necrotic tissue needs to be removed with surgical debridement. MDT Diabetes roles of different professionals Doctor –

• • •

Consultant - Diagnose condition manage drugs, give health advise, oversee care, performs annual review, manages severe acute hypo/hyper glycemic incidents GP – takes care of day to treatment, prescription resissues any non urgent problems, controls referral to secondary care, looks after associated risk factors Other specialist eg cardiologists, vascular surgeons, neurologists, opthalmologists may be involved if complications occur

Nurse • Specialist diabetic nurse – provides support in treatment regimes, insures good technique and first point of contact with specialist team • Practise nurse – check bp, advises on health issue supports the specialist team in the community • District nurse – may visit elderly or infirm patients in their home to support their care Other professions • Opticians/ optical technicians – performs check in the health of the diabetic eye, check for retinopathy • Podiatrist - maintains health of feet, check for neuropathy and cuts nails protects from ulcers etc • Dietician – helps draw up a suitable diet with regards to sugar intake, GI foods and nutrition Infections in the immune compromised patient (diabetic special) • • •

• •

• • • •





Immunocompromised patients have immunity changes that increase both the risk of infection and the ability to combat infection Immunity may be impaired temporarily or permanently as a result of either an immunodeficiency disease state (congenital or acquired) or induced immunosuppression i.e. cancer therapy or transplants Risk varies according to degree of immunosuppression inversely o High-risk:  Haematological malignancies  AIDS patients with low CD4+ counts  Bone marrow transplantation  Splenectomy  Genetic disorders such as severe combined immunodeficiency. o Intermediate-risk:  Solid tumours (particularly after cytotoxic chemotherapy)  HIV/AIDS  Solid organ transplant. o Low-risk:  Long-term corticosteroid use (such as patients with rheumatoid arthritis)  Patients with areas of locally reduced immune function e.g. lymphodema due to stasis  Diabetics profoundly neutropenic asplenic or have a dysfunctional spleen are at special risk and infections in these patients should be treated as a medical emergency A very wide range of organisms can cause infection in immunocompromised patients. The source of the infection may be exogenous or endogenous and is very difficult to determine Commensals such as Candida and other fungi, and viruses such as cytomegalovirus, can lead to serious infection also Opportunistic pathogens must not be overlooked. Broad-spectrum antibiotic use increases the risk of secondary fungal infections. Gram-positive organisms are of greater risk. Diabetic patients are susceptible to infection due to the short-term influence of hyperglycaemia on host defence mechanisms, particularly on neutrophil function and the arterial system. Diabetic patients with poor glycemic control may have abnormalities in lymphocyte number and function Fever is often the only symptom of infection in the immunocompromised patient and always requires further investigation. No pathognomonic pattern or degree of fever can be associated with specific infection and presentation is often atypical

  

   

if fever of >38°C persists for 2 h or more, broad-spectrum antibiotic therapy should be administered intravenously (high or intermediate risk patients) Due to the difficulty in isolating the source and causative organism a battery of tests is usually necessary – Blood cultures, wound swabs, urine sample (if UTI suspected), sputum sample, possibly CSF all for culture and sensitivity Until an empirical treatment can start – t he core regimen should include: A combination of broad-spectrum antibiotics at high-doses to combat Gram-positive and Gram-negative aerobes, plus antifungal therapy from the outset of treatment to prevent secondary fungal infection IV for rapid onset of action Consideration of local factors ie. underlying disease state, presence of an intravascular device, local bacterial ecology and known resistance patterns. If fever persists and no pathogen is isolated it is possible that a viral or fungal infection is present. non-infective conditions such as graft-versus-host disease or thrombosis should be considered

Local policy for immunocompromised patients is Tazocin® 4.5g tds IV AND Gentamicin# 5mg/kg IV (single daily dose regimen) Max 480mg If septic shock (hypotensive, tachycardic, vasodilated, tachypnoea) add in: Vancomycin *1g od IV (or Teicoplanin 400mg bd IV for 3 doses then 400mg od thereafter)

Diabetes testing for – Urine testingSimple dipstick for glucose • Commonly done – ideally on a void 1-2 hours after a meal • Glycosuria should always be followed by blood testing • Glycosuria is not diagnostic of diabetes – false positives include ~ low renal threshold, pregnancy, common in young healthy adults Ketone testing• Ketonuria can indicate someone who is in ketotic phase of diabetic ketoacidosis • False positives include fasting, strenuous exercise, high fat low carb (atkins) diet Protienuria• Dipstick testing of albumin is used for detecting renal disease at a threshold of 300mg/l • Microalbuinaria can be seen as risk factor for diabetic nephropathy Blood testingFingerprick testing is less accurate then a full blood sample as• Glucose concentrations are lower in venous then arterial blood • Whole blood concentrations lower then plasma levels Diabetes is diagnosed on the basis of – • Fasting plasma glucose ≥7.0 mmol/l – 6.1 -7 you need to complete a OGTT • Random plasma glucose≥ 11.1mmol/l – 7.8 – 11.0 you need to complete a OGTT Oral Glucose Tolerance test (OGTT) • Unrestricted diet for 3 days, fasted over night (8hrs) • Rest for 30 mins prior to test, no smoking plasma glucose measured at start, administer glucose 75g then two hours after

Venous Plasma

Venous whole blood

Capillary plasma

Capillary whole

Diabetes Fasting 2hrs after glucose Impaired tolerance Fasting 2 hrs after glucose

mmmol/l

mmmol/l

mmmol/l

blood mmmol/l

≥ 7.0 ≥ 11.1

≥ 6.1 ≥ 10.0

≥ 7.0 ≥ 12.2

≥6.1 ≥11.1

< 7.0 7.8 -11.0

<6.1 6.7 – 9.9

<7.0 8.9 – 12.1

< 6.1 7.8 – 11.0

HB1AC• Glycated haemoglobin provides an accurate of glycaemic control over weeks to months • HB1AC glucose ratio in comparison to HBA0 shows how much glucose is present • 1% rise in HB1AC equals 2mmol/l in blood glucose • Ideal HB1AC in Diabetics is <7.5 low risk <6.5 in high risk (arterial risk) BMI

• • • • • •



Body mass index is defined as body weight divided by the square of their height in metres (kg/m2) Divised by a Belgian in the 1850’s it did not come into common use till the 1950’s and 60’s. As a simple numeric measure of fatness and thinness The idea has since been leapt upon insurance companies as risk index Advantages it is quick and easy to use, the formula is simple, gives a numerical value to subjective opinion Disadvantages assumptions about distribution of muscle and bone mass, overestimates adiposity on those with more lean body mass (e.g. athletes) while underestimating adiposity on those with less lean body mass (e.g. the elderly) BMI ranges o 18.5 to 24.9 – healthy weight. o 25 to 29.9 – overweight. o 30 or more – obese. Other measures of Obesity include – body fat percentage, waist to hip ratio with their own benefits and drawbacks

ObesityObesity has been shown to be a risk factor for – • Heart disease, CVA/TIA, Artherosclerosis, Diabetes, Vascular disease, Osteoarthritis, some cancers • Figures vary wildly but 21 %(NS0 2002) of adults are classed as oobese and the trend is increasing • Disturbingly the trend for obese children show increases of 3% in recent years (national statistics office) 2002 27% of kids 2 -18 overweight, 7% obese (BMI 30+) • Age is a risk factor for obesity so overweight kids are obese adults – older you get fatter you get !!! Anatomy of the Bowmans capsule, nephron

• • • • • •







• • • •

A nephron is the smallest functional unit in the kidney It conssits of – Bowmans capsule, Glomerulus –imagine a fist pushed into a ballon the ballon being the capsule the fist the glomerulus The function of the nephron is to filter the blood from the efferent areteriole There are several layers of cells which filter all the blood, creating an ultrafiltrate Passing from inside the blood vessel towards collecting duct the layers areEndothelial cell layer – o the walls of the glomerulus(ateriole) contain numerous pores (fenestrae), that are un diaphragm. o openings which are so large that anything smaller than a red blood cell passes through that layer. o endothelial cells lining the glomerulus are not usually considered part of the renal filtration barrier as they filter so little Glomerular basement membrane o The glomerular endothelium sits on a very thick (250-350 nm) glomerular basement membrane. o This membrane is thick compared to most other basement membranes (40-60 nm), but it is also rich in negatively charged glycosaminoglycans such as heparan sulfate. o The negatively-charged basement membrane repels negatively-charged proteins from the blood, helping to prevent their passage into Bowman's space. Podocytes o Podocytes line the other side of the glomerular basement membrane. o They form a tight interdigitating network of pedicels that control the filtration of proteins from the capillary lumen into Bowman's space. o The space between adjacent podocyte is spanned by a slit diaphragm formed by several proteins including podocin and nephrin. o Podocytes have a negatively-charged coat (glycocalyx) that limits the filtration of negativelycharged molecules, such as serum albumin. o podocytes are sometimes considered the "visceral layer of Bowman's capsule", rather than part of the glomerulus normal rate of filtration is 125 ml/min, equivalent to ten times the blood volume daily. Anything under roughly 30 kilodaltons can pass freely through the membrane. Extra hindrance for negatively charged molecules due to the negative charge of the basement membrane and the podocytes keeps albumin and proteins in the blood small molecules such as water, glucose, salt (NaCl), amino acids, and urea pass freely into Bowman's space, but cells, platelets and large proteins do not. Ultrafiltrate leaving the Bowman's capsule is very similar to blood plasma in composition as it passes into the proximal convoluted tubule.

Chronic Kidney disease – • Chronic renal failure is the progressive loss of nephrons resulting in permanent compromise of renal function it is divided into stages based on eGFR as follows

• •

o

0 Normal kidney function – GFR above 90ml/min/1.73m2 and no evidence of Kidney damage*

o

CKD1 – GFR above 90ml/min/1.73m2 with evidence of Kidney damage

o

CKD2 (Mild) – GFR above 60 to 89 ml/min/1.73m2 with evidence of Kidney damage

o

CKD3 (Moderate) – GFR above 30 to 59 ml/min/1.73m2

o

CKD4 (Severe) – GFR above 15 to 29 ml/min/1.73m2

o

CKD5 Kidney failure (dialysis or kidney transplant needed) – GFR less than 15 ml/min/1.73m2

o

*persistent microalbuminuria, persistent proteinuria, persistent haematuria (after exclusion of other

causes, e.g. urological disease), structural abnormalities of the kidneys(US SCAN), Presentation of chronic kidney disease can include – nausea, anorexia, vomiting, Diarrhoea, amenorrhoea, erectile impotence, infertility, uraemic pericarditis, hypertension, peripheral vascular disease, heart failure, confusion, coma, fits ……. The length of the list shows that manifestations are widespread and renal failure is often an incidental finding a product of age or other systemic illness



Common causes o glomerulonephritis - accounts for 25% of cases o multisystem disease  diabetes mellitus is the main member of this group  Alport's syndrome o acute pyelonephritis / tubulointerstitial disease o hypertension and vascular causes o polycystic kidney disease - the most common cause of familial chronic renal failure o idiopathic in 15% of cases

Renal function tests- Investigations include





• • •

urine o o o o o Blood o o o o

microscopy, culture and sensitivity sodium, urea, creatinine and osmolality proteinuria haematuria and proteinuria Red cell casts suggest glomerulonephritis; tubular cell casts, acute tubular necrosis

FBC – checking for anaemia, clotting and signs of infection ESR – indication of an active inflammatory processes urea and electrolytes – checking for imbalances measure of renal function Creatinine – measure of the renal function Creatinine is produced at a regular rate in most people it’s clearance is a sign of the functionality of the kidneys o LFT – check for associated pathology in the liver o CRP – indication of an active inflammatory proccess ECG - may reveal changes associated with hyperkalaemia Imaging – o Ultrasound - assess renal size ?evidence of obstruction o CXR - ?evidence of pulmonary oedema end stage renal failure Renal biopsy – to differentiate the neuropathies

Related Documents

Scenario Three
November 2019 11
Scenario Three
November 2019 2
Scenario
October 2019 65
Scenario
October 2019 70
Scenario
May 2020 42
Scenario
July 2020 38